Sei sulla pagina 1di 71

Page 1 of 71

Biochemistry

BIOCHEMISTRY

1. Which of the following single-base DNA gene coding changes is least likely to significantly
affect the function of resulting protein? (Harper pages 361-363)

A. Missence mutation resulting to histidine-to-glutamic acid change


B. Silent mutations in the third codon base
C. Frameshift mutation
D. Nonsense mutation

2. Which of the following is a property of enhancer elements? (Harper pages 384-385)

A. They do not work located downstream of the promoter.


B. They could work when located long distances from the promoter.
C. Upon activation, they could repress transcriptional rate.
D. They are proteins binding to DNA sequences.

3. Which of the following is considered as a transcriptional regulation control? (Harper pages


383)

A. Chromatin remodelling
B. Alternative splicing
C. RNA editing
D. Capping of the mRNA

4. Which of the following is true of biological membrane structure? (Harper pages 415-421)

A. Saturated fatty acids increase fluidity.


B. Cholesterol is an amphipathic molecule.
C. The inner and outer sheets are symmetrical to each other.
D. They contain transport proteins, which are mainly peripheral.

5. Which of the following defines the specificity of hormone activity in a cell? (Harper page
435)

A. The synthesis of the hormone in the target cell


B. The presence of the hormone receptor
C. The upregulation of the hormone receptor in a cell
D. Absence of other target cells adjacent to hormone-producing cells

6. Which of the following is true of hormones? (Harper pages 437-38)

A. Hormones with intracellular receptors are usually hydrophilic.


B. Lipophilic hormones need transport proteins.
C. Cathecholamines and steroid hormones act via a plasma membrane receptor.
D. Steroid hormones are derivatives of amino acids.

7. Which vitamin is essential for one-carbon metabolism? (Harper page 494)

A. Thiamine
B. Riboflavin
C. Niacine
D. Folic acid

8. Which of the following correctly characterize a component of the extracellular matrix?


(Harper 535-541)

A. Collagen type IV, a component of the basement membrane, form compact bundles.
B. Elastin is essential for cell-cell communication.
C. Fibronectin is involved mainly in cell adhesion and signaling.
D. Laminin acts intracellularly as a major component of the cytoskeleton.

9. Which of the following is NOT correctly paired? (Harper page 593)

Page 1 of 71
Page 2 of 71
Biochemistry

A. IgA: found in secretions


B. IgM: transplacental passage
C. IgG: opsonization
D. IgE: allergic responses

10. Which of the following statements characterizes ATP? (Harper page 84)

A. It is used in the synthesis of DNA.


B. It contains 3 phosphate bonds for energy storage.
C. It is used to drive thermodynamically unfavorable reactions.
D. It is produced mainly by substrate level phosphorylation.

11. Enzymes, as biological catalysts accelerate velocity of reactions by:


A. lowering the free energy of activation of reactions
B. increasing transition state of reactants and products
C. lowering the net energy change between initial and final states of the reaction
D. increasing the equilibrium constant (Keq) of the reaction

12. A general class of enzymes that join molecules by creating a new chemical bond at the
expense of ATP hydrolysis
A. lyase
B. isomerase
C. ligase
D. hydrolase

13. An enzyme catalyzing the reaction: lecithin + cholesterol -- fatty acid cholesterol ester +
lysolecithin is classified as:
A. oxidoreductase
B. lipase
C. transferase
D. hydrolase

14. The Michaelis Menten constant is equal to substrate concentration when:


initial velocity is ½ Vmax
Vmax is doubled
Km= ½ substrate concentration
Km/Vmax = 1

15. The kinetic parameter that reflects the affinity of the enzyme for its substrate is called:
A. Vmax
B. Q10
C. Km
D. Kcat

16. Double reciprocal plot of the Michaelis Menten equation is called:


A. Hanes-Wolf
B. Eadie Hofstee
C. Lineweaver Burk
D. Briggs-Haldane

17. In the Lineweaver Burk plot, the slope of the line is equal to:
A. -1/Km
B. Km/Vmax
C. 1/Vmax
D. 1/S

18. Which statement is true about competitive inhibition?


A. Inhibitor binds to the allosteric site of the enzyme
B. Km is constant while Vmax decreases
C. Usually the inhibitor structurally resembles the normal substrate
D. Both the Km and Vmax decrease

19. Serum enzymes are elevated following tissue injury and death due to:

Page 2 of 71
Page 3 of 71
Biochemistry

A. enzyme induction secondary to stimulation of protein synthesizing machinery of the


cell
B. allosteric enzyme activation by positive activators
C. increased cell membrane permeability and leakage of enzymes into blood
D. proteolytic activation of proenzymes or zymogens

20. Which of the following coenzymes is not appropriately paired with the enzymatic reaction
it is associated with?
E. tetrahydrofolate- transamination
F. biocytin-carboxylation
G. NAD- dehydrogenation
H. Coenzyme A- transacylation

21. The active form of Vitamin B12 coenzyme:


A. 5’-deoxyadenosylcobalamin
B. hydrocobalamin
C. aquocobalamin
D. cobamide

22. In Bioenergetics, the amount of energy available for the performance of useful work is
termed:
A. entropy
B. free energy
C. enthalpy
D. equilibrium energy

23. During biological oxidations, reduced coenzymes such as NADH and FADH 2 generate energy
when oxidized thru the:
A. Krebs Cycle
B. Electron Transport Chain
C. Cytochrome P450 system
D. Embden Meyerhof Pathway

24. The final acceptor of electrons in the Respiratory Chain is:


A. cytochrome oxidase
B. coenzyme Q
C. iron sulfur center
D. oxygen

25. Substrate level phosphorylation differs from oxidative phosphorylation in that:


A. oxidative phosphorylation synthesizes ATP via the Electron Transport Chain
B. substrate level phosphorylation only takes place in the cytosol
C. oxidative phosphorylation is coupled to a direct transfer of high energy ~Pi from a high
energy compound to ADP
D. Substrate level phosphorylation is more energy efficient per molecule of substrate
oxidized

26. Which of the following affects biological oxidation by dissociating mitochondrial respiration
from phosphorylation?
A. 2,4 dinitrophenol
B. B. rotenone
C. C. antimycin
D. cyanide

27. Which of the following statements is correct regarding digestion of carbohydrates?


A. Digestion of dietary polysaccharides takes place to a significant extent in the stomach
due to acid hydrolysis.
B. Digestion initially takes place in the mouth through the action of - amylase.
C. Pancreatic amylase hydrolyzes -1,4 glycosidic linkages in cellulose.
D. The final end products of digestion are all glucose molecules from dietary
carbohydrates

28. In man, glucose from dietary cellulose could not be available for energy because:

Page 3 of 71
Page 4 of 71
Biochemistry

A. cellulose is made up of very long polysaccharide chain made up of  1,4 and  1,6
glycosidic bonds
B. human diet does not have a significant amount of cellulose
C. man inherently lacks enzyme to hydrolyze cellulose
D. cellulose is not made up of glucose units

29. A 3 month old infant develops diarrhea and abdominal discomfort a following a milk
formula. However feeding history revealed that he was able to tolerate breast feeding and
the same milk formula at birth. The most likely condition is
A. Primary - galactosidase deficiency
B. Milk allergy
C. Lactose malabsorption
D. Secondary lactose intolerance

30. The major pathway of glucose metabolism that exists in all cells is the:
A. Pentose Phosphate Pathway
B. Citric Acid Cycle
C. Uronic Acid Pathway
D. Embden Meyerhof Pathway

31. Hexokinase differs from glucokinase in which aspect?


A. Glukokinase has a higher affinity to glucose
B. Hexokinase is induced by insulin
C. Glucokinase phosphorylates glucose to glucose 6 phosphate
D. Hexokinase is inhibited by glucose 6 phosphate

32. The end product of glycolysis in exercising muscle and red blood cells is:
A. pyruvic acid
B. acetic acid
C. lactic acid
D. acetoacetic acid

33. The most important positive regulator of the committed step of glycolysis is the availability
of
A. fructose 1,6 bisphosphate
B. ATP
C. Citrate
D. fructose 2,6 bisphosphate

34. True of the The Malate Aspartate Shuttle


A. can be considered as an anaplerotic reaction
B. less energetically efficient than the  glycerophoshate shuttle
C. transports NADH from the cytosol into the mitochondria for oxidative
phosphorylation
D. provides reducing equivalents for reductive biosynthesis of fats

35. The initial step in the TCA Cycle is catalyzed by:


A. pyruvate dehydrogenase
B. lactate dehydrogenase
C. citrate synthase
D. -keto-glutarate dehydrogenase

36. The Hexose Monophosphate Shunt is metabolically significant to nucleic acid metabolism
because it:
A. provides NADPH
B. furnishes pentose phosphates
C. is an alternative pathway for glycolysis
D. stabilizes structure of DNA

37. The rate limiting step in the Pentose Phosphate Pathway is catalyzed by:
A. hexokinase
B. glucose-6 phosphate dehydrogenase
C. 6- phosphogluconolactonase
D. transketolase

Page 4 of 71
Page 5 of 71
Biochemistry

38. Which of the following enzymatic reaction is common to both glycolysis and
gluconeogenesis?
A. hexokinase
B. pyruvate carboxylase
C. phosphofructokinase 1
D. phosphotriose isomerase

39. The inherent absence of this enzyme prevents muscle glycogenolysis to maintain glucose
homeostasis:
A. glycogen phosphorylase
B. glucose- 1- phosphatase
C. glucose-6-phosphatase
D. UDP-Glucose pyrophosphorylase

40. Glycogen synthesis is activated while glycogenolysis is inhibited when:


A. glucagon/ insulin ratio is elevated
B. glycogen synthase is covalently modified by phosphorylation
C. cyclic AMP dependent protein kinase is activated
D. glycogen synthase I predominates over glycogen synthase D

41. Mannose and Fructose enter the mainstream of carbohydrate metabolism via:
A. Citric Acid Cycle
B. Embden Meyerhoff Pathway
C. Hexose Monophosphate Shunt
D. Glyoxylic Acid Cycle

42. Fatty acids enter the mitochondrial matrix for oxidation in what form?
A. acyl-CoAs through active transport
B. as free fatty acids
C. following conversion to carnitine
D. complexed with carnitine as acyl-carnitine

43. The number of reduced electron carriers generated when one mole of palmitate is
subjected to oxidation:
A. 2 FADH2 and 3 NADH + H+
B. 3 FADH2 and 2 NADH + H+
C. 7 FADH2 and 7 NADH + H+
D. 5 FADH2 and 5 NADH + H+

44. Majority of the reducing equivalents necessary for fatty acid biosynthesis are generated
during conversion of:
A. 1,3 bisphosphoglycerate to 3 phosphoglycerate
B. pyruvate to malate
C. glucose 6-phosphate to 6 phosphogluconate
D. 6 phosphogluconate to ribulose 5-phosphate

45. How many moles of NADPH + H+ can be derived from the pathway utilized in the transport
of acetyl CoA out of the mitochondria?
A. 8 moles
B. 10 moles
C. 12 moles
D. 14 moles

46. The site of action of the NSAIDS is:


A. thromboxan synthase
B. prostaglandin cyclooxygenase
C. lipoxygenase
D. prostacyclin synthase

47. Hepatocytes contribute to the overall body content of ketone bodies primarily because of
the absence of:
A. β-ketothiolase necessary to hydrolyze acetoacetyl CoA
B. HMG-CoA lyase

Page 5 of 71
Page 6 of 71
Biochemistry

C. HMG-CoA synthetase
D. 3-ketoacyl-CoA transferase or acetoacetate succinyl CoA transferase

48. The principal plasma carrier of dietary triacylglycerols is:


A. serum albumin
B. LDLs
C. HDLs
D. chylomicrons

49. Activation of lipoprotein lipase requires which apoprotein?


A. Apo E
B. Apo B-100
C. Apo C-II
D. Apo B-48

50. VLDLs are synthesized in the liver for transport of which of the following to the peripheral
tissues?
A. triacylglycerols
B. apoproteins for HDL synthesis
C. LDL receptors
D. apo B-100

51. The committed step of cholesterol biosynthesis is catalyzed by this enzyme:


A. squalene synthase
B. HMG CoA reductase
C. mevalonate kinse
D. HMG CoA lyase

52. High dietary intake of cholesterol leads to:


A. increased HMG CoA reductase synthesis
B. decreased apo B-100 synthesis
C. decreased ACAT activity
D. decreased LDL receptor synthesis

53. Several of the enzymes of cholesterol biosynthesis require which of the following cofactors?
A. biotin
B. NADPH
C. FADH2
D. TPP

54. The molecule that helps stabilize the action of pancreatic lipase on dietary lipids:
A. colipase
B. chenodeoxycholic acid
C. cholic acid
D. enteropeptidase

55. Source of the nitrogen atoms required for animal metabolism can be directly obtained from:
A. atmospheric nitrogen
B. NO2
C. ammonia
D. nitric oxide

56. Aside from urea, the main circulatory system carrier of nitrogen atoms destined for
excretion by the kidney is:
A. glutamate
B. asparagine
C. alanine
D. glutamine

57. A deficiency in the ability to secrete enterokinase leads to which of the following?
A. abnormally low levels of intestinal zymogens
B. abnormally low levels of active intestinal proteases
C. abnormally low levels of undigested proteins in lower GIT
D. increase plasma levels of serum amylase

Page 6 of 71
Page 7 of 71
Biochemistry

58. Transamidation reaction include which of the following?


A. formation of liver glutamate from α-ketoglutarate
B. formation of liver glutamine from glutamate
C. formation of liver asparagine from aspartate
D. formation of liver oxaloacetate from alanine

59. The important cofactor for the transaminases:


A. pyridoxal phosphate
B. tetrahydrofolate
C. biocytin
D. tetrahydrobiopterin

60. The production of alanine by peripheral tissue, especially by the muscle, is primarily
dependent on:
A. allosteric effectors of ALT
B. intracellular levels of pyruvate
C. intracellular levels of glutamate
D. intracellular levels of NADH

61. Nitrogen of muscle alanine can be derived from tissue ammonia through this enzyme
action:
A. glutamate dehydrogenase
B. alanine dehydrogenase
C. glutamine synthetase
D. alanine synthetase

62. Sulfur for cysteine synthesis is derived from :


A. sulfites
B. H2S
C. Coenzyme A
D. methionine

63. Severe PKU and shortened life span is a result of:


A. excess dietary phenylalanine
B. excess dietary tyrosine
C. deficiency of dihydrobiopterin reductase
D. deficiency of tetrahydrofolate

64. Serine is synthesized by:


A. transamination of an amino acid with α-ketoglutarate as an amino acceptor
B. transamination of an α-ketoacid derived from TCA
C. transamination of an α-ketoacid derived from EMP
D. modification of an essential amino acid

65. Lysine is an essential amino acid because:


A. animals cannot make the lysine carbon skeleton
B. the ε-amino group of lysine cannot be formed by an aminotransferase
C. the α-carboxylic cannot be added to urocanate
D. lysine cannot be subjected to deamidation

66. Amino acids that are strictly ketogenic:


A. lysine and leucine
B. leucine and isoleucine
C. valine and isoleucine
D. tyrosine and tryptophan

67. Choose the correct precursor-product relationship from the following:


A. tyrosine is a precursor to dihydroxyphenylalanine
B. tyrosine is a precursor to melatonin
C. melatonin is a precursor of serotonin
D. dopamine is a precursor to norepinephrine

68. The Watson-Crick model or the B-form of DNA is typified by:

Page 7 of 71
Page 8 of 71
Biochemistry

A. 12 base pairs per helical turn


B. a smaller number of purines than pyrimidines
C. bases located in the interior of the right-handed helix
D. two single strands that run in the 5’ 3’ direction

69. Guanine-cytosine (G-C) base pairs in a DNA molecule are more stable to denaturation than
adenine-thymine (A-T) pairs because:
A. adenine and thymine have bulkier structures than guanine and cytosine
B. A-T base pairs exhibit less electron clouds around them
C. G-C base pairs contain more hydrogen bonds than A-T pairs
D. the phosphodiester bonds among G-C base pairs are weaker

70. DNA is distinguished from RNA by:


A. a high content of modified bases
B. its component sugar, ribose
C. its single stranded structure
D. the presence of thymine instead of uracil

71. Anticancer drugs like 5-fluorouracil and 6-mercaptopurine act against malignant cells by:
A. being incorporated into DNA before replication
B. causing the denaturation of DNA in cancer cells
C. inactivating protein synthesis
D. inhibiting the enzymes that synthesize thymidylic acid

72. Which of the following is a nucleotide?


A. uracil
B. cytidine
C. guanosine
D. adenylic acid

73. Which vitamin is essential in the synthesis of purine nucleotides?


A. ascorbic acid
B. folic acid
C. niacin
D. thiamin

74. Which clinical condition does not involve a defect in purine metabolism?
A. adenosine deaminase deficiency
B. gout
C. Lesch-Nyhan syndrome
D. orotic aciduria

75. Salvage reactions that generate purine and pyrimidine nucleosides and nucleotides are
characterized as:
A. involving the same enzymes as in de novo synthesis
B. more efficient than nucleotide synthesis from new precursors
C. providing greater output of purines and pyrimidines
D. requiring less energy than does the de novo synthesis

76. The enzyme that catalyzes the committed step of purine nucleotide de novo pathway:
A. carbamoyl phosphate synthetase I
B. glutamine amidophosphoribosyl transferase
C. carbamoyl phosphate synthetase II
D. aspartate transcarbamoylase

77. In the management of gout, allopurinol is given to inhibit xanthine oxidase, the enzyme that
catalyzes the synthesis of:
A. allantoate
B. ammonia
C. urea
D. uric acid

78. The correct flow of genetic information according to the Central Dogma of Molecular
Biology is:

Page 8 of 71
Page 9 of 71
Biochemistry

A. replication -----> transcription ----> translation


B. replication -----> translation ----> transcription
C. transcription ---- > replication ----> transcription
D. translation ----> replication ----> transcription

79. Transfer RNA (tRNA) as an adaptor molecule serves to:


A. carry the nucleotide sequences that will dictate the amino acid to be synthesized
B. hold the promoter site that will be recognized by the polymerase
C. recognize the triplet codon in mRNA via its anti-codon arm
D. synthesize the amino acid to be incorporated into the peptide

80. Degeneracy of the Genetic Code means that:


A. an amino acid can only be decoded by a single codon
B. a single codon encodes only one amino acid
C. multiple codons may encode the same amino acid
D. with time, codons degenerate into senseless code

81. The final product of gene expression is:


A. amino acid
B. nucleotide
C. polynucleotide
D. protein

82. Normal growth and development of cells are controlled by genes known as:
A. leader genes
B. non-coding genes
C. proto-oncogenes
D. transforming genes

83. The type of mutation that involves the substitution of a purine base with another purine
base is:
A. transition
B. transposition
C. transpyrimidation
D. transversion

84. This mineral helps guard against DNA damage as a cofactor of glutathione:
A. iron
B. manganese
C. selenium
D. zinc

85. A tool in molecular biology in which a DNA sequence of interest is exponentially amplified in
vitro by repeated cycles of enzymatic DNA synthesis is:
A. DNA sequencing
B. in situ hybridization
C. Northern blotting
D. polymerase chain reaction

86. In the construction of recombinant DNA, which enzyme is used to cut DNA at precise base
sequences?
A. DNA polymerase
B. DNA-RNA exonuclease
C. restriction endonuclease
D. T4 DNA ligase

87. In recombinant DNA technology, antibiotic resistance genes in plasmids are used to:
A. control copy number in bacteria
B. facilitate transformation of host cells
C. provide a marker for start of replication
D. select and screen for recombinant DNA clones

88. The pain felt by a patient with an enlarging malignant tumor is partly contributed by the:
A. exaggerated metabolism of nutrients by the tumor cells

Page 9 of 71
Page 10 of 71
Biochemistry

B. loss of enzymes for glucose oxidation in cancer tissue


C. release of cytokines such as IL-1 and IL-6 by the tumor cells
D. overgrowth of blood vessels in the interior of the cancer mass

89. Malignancy may be naturally prevented if the mutated cancer gene was repaired:
A. after gene expression
B. before DNA replication
C. following transcription
D. shortly after DNA synthesis

90. With persistence of a mutation in cells during organogenesis, the resulting event is:
A. development of cancer
B. inactivation of genetic disorders
C. occurrence of physical malformations
D. promotion of sterility

91. To specifically identify the mutation in the affected DNA segment in a patient with cancer,
which blot transfer method should be used?
A. Northern
B. Southern
C. Southwestern
D. Western

92. The enzyme, Taq polymerase is used in Polymerase Chain Reaction (PCR) because it:
A. facilitates annealing of primers
B. is resistant to heat denaturation
C. promotes DNA polymerization
D. provides optimum condition for DNA synthesis

93. A 60-year-old woman has developed red, roughened skin in sun-exposed areas over the
past 2 years. She also has a chronic, watery diarrhea. On physical examination she exhibits
memory loss with confusion. These findings are most consistent with which of the following
vitamin deficiencies? PELAGRA
A. Thiamine
B. Pyridoxine
C. Niacin
D. Vitamin A

94. A clinical study is performed involving dietary iron metabolism in adults. It is observed that
intestinal absorption of iron can be enhanced in patients with iron deficiency anemia by
supplementing their diet with another nutrient. Which of the following vitamins is most
likely to have this effect?
A. D
B. E
C. C
D. B3

95. A 6-year-old child was born at term, with no congenital anomalies. She is now only 70% of
normal body weight, though she shows dependent edema of the lower extremities as well
as an enlarged abdomen. Her flaking skin shows irregular areas of depigmentation,
hyperpigmentation, and desquamation. These findings are most suggestive of which of the
following nutritional problems?
A. Marasmus
B. Kwashiorkor
C. Niacin deficiency
D. Vitamin A Deficiency

96. A 41-year-old man has had increasing dyspnea for the past year. On physical examination he
has diffuse crackles at lung bases. A chest x-ray shows pulmonary edema and cardiomegaly.

Page 10 of 71
Page 11 of 71
Biochemistry

Echocardiography shows an ejection fraction of 40%. Laboratory studies show hemoglobin


14 g/dL, hematocrit 42%, and WBC count 8320/microliter. A deficiency in which of the
following vitamins is most likely to produce these findings? WET BERI BERI
A. A
B. B1
C. B2
D. K

97. A 7-year-old child has complained of pain in his legs for the past year. On physical
examination, there is bowing deformity of his lower extremities. Plain film radiographs of his
lower legs shows widened epiphyses and bowing of tibiae. Bone mineral density appears
normal, consistent with failure of osteoid matrix formation. Which of the following vitamin
deficiencies is this child most likely to have?
A. D
B. E
C. C
D. B3

98. A 28-year-old man has a history of multiple and recurrent pulmonary infections since
childhood. He also has noted foul smelling stools for the past 10 years. Laboratory studies
show an elevated sweat chloride test. He has a quantitative stool fat of 10 g/day. A
deficiency state involving which of the following nutrients is most likely to develop in this
patient?
A. Vitamin B1
B. Vitamin D
C. Calcium
D. Vitamin A

99. A clinical study is performed to compare the risk for health problems in obese persons with
a BMI OF 35, compared with a control group of persons with a BMI between 20 and 25.
Persons in these two groups are followed for 20 years. Which of the following conditions is
most likely to appear equally in both groups?
A. Cholelithiasis
B. Cirrhosis
C. Hypertension
D. Diabetes mellitus

100. A 15-year-old girl has been under a physician's care for the past year after diagnosis of
anorexia nervosa. Her BMI is now 18. On physical examination she has cheilosis. Laboratory
studies show hemoglobin 13.7 g/dL, hematocrit 41.0%, MCV 88 fL, platelet count
191,055/microliter, and WBC count 4930/microliter. Her serum glucose is 66 mg/dL. Which
of the following nutrient deficiencies is most likely to cause this patient's findings?
A. Riboflavin
B. Vitamin C
C. Folic Acid
D. Iron

101. The metabolic function of the pentose phosphate pathway is to:


A. act as a source of ADP biosynthesis.
B. generate NADPH and pentoses for the biosynthesis of fatty acids and nucleic acids.
C. participate in oxidation-reduction reactions during the formation of H 2O.
D. provide intermediates for the citric acid cycle.

102. The FNRI Philippine RENI Committee 2002 recommends the following protein intake:

Page 11 of 71
Page 12 of 71
Biochemistry

A. 1.14 g/kg DBW/day


B. 2.14 g/kg DBW/day
C. 3.14 g/kg DBW/day
D. 4.14 g/kg DBW/day

103. Number of moles of ATP generated from the complete oxidation of pyruvate as it undergoes
the pyruvate dehydrogenase reaction and the Kreb’s cycle is:
a. 12 c. 14
b. 13 d. 15
Page 113 Harper’s 26th Edition Chapter 16

104. Which of the following enzymes when deficient will result in the accumulation of
gangliosides in patients suffering for Tay-Sach’s Disease?
a. glucocerebrosidase c.sphingomyelinase
b. aryl sulfatase d.hexosaminidase
Page 203 Harper’s 26th Edition Chapter 24

105. Which of these is deficient in Cori’s disease?


a. glucose-6-phosphate c. branching enzyme
b. glycogen phosphorylase d.debranching enzyme
Page 152 Harper’s 26th Edition Chapter 18

106. Just before breakfast, what is the source of glucose in the body?
a. glucose c. ketones
b. glycogen d. fatty acids
Page 102,107-108 Harper’s 26th Edition Chapter 13

107. For every mole of glucose that goes through the glycolytic pathway, what is the net number
of pyruvate generated?
a. one c. three
b. two d. four
Page 136-144 Harper’s 26th Edition Chapter 17

108. Which of these is the primaryfunction of the Pentose Phosphate Pathway?


a. generation ATP
b. NADPH generation
c. NADPH generation and production of ribose residues
d. Provision of ribose residues of nucleotide and nucleic acid synthesis
Page 163-168 Harper’s 26th Edition Chapter 20

109. Which enzyme generates NADPH + H needed to reduce GSSG:


a. glutathione reductase c. aldehyde dehyrogenase
b. glucose 6PO4 dehydrogenase d. lactate dehydrogenase
Page 156-165 Harper’s 26th Edition Chapter 19

110. Glucose 6 phosphate dehydrogense deficiency is characterized by which one of the


following?
a. accumulation of glucose -6-phosphate in the tissue
b. decreased formation of NADPH
c. increased synthesis of reduced glutathione
d. may manifest as hemorrhagic anemia
Page 169-170 Harper’s 26th Edition Chapter 20

111. The Citric acid cycle as the final common pathway in fuel metabolism provides greatest
amount of energy by being directly linked to:
a. glycerophosphate shuttle c. glycolysis
b. malate aspartate shuttle d. electron transport chain
Page 130-133 Harper’s 26th Edition Chapter 16

112. Cyanide poisoning is very fatal. This is bacuse cyanide has this effect of the Electron
Transport Chain:
a. inhibits complex III
b. inhibits cytochrome oxidase

Page 12 of 71
Page 13 of 71
Biochemistry

c. uncouples oxidative phosphorylation


d. enhance the transport of electrons
Page 92 Harper’s 26th Edition Chapter 12

113. At high concentration of blood glucose, as in uncontrolled diabetes mellitus, hemoglobin


reacts with glucose to form this compound which can be assayed to monitor glycemic
control
a. methemoglobin
b. hemochrome
c. glycosylated hemoglobin
d. hemin
Page 47 Harper’s 26th Edition Chapter 6

114. The precursor of the sugar-phosphate moieties in the de novo synthesis of both purine and
primidine nucleotide is:
a. glucose 1 phosphate
b. glucose 6 phosphate
c. ribose 1 phosphate
d. ribose 5 phosphate
Page 286-292 Harper’s 26th Edition Chapter 33

115. The highest reservior of calories in muscle tissues is:


a. Fats c. glycogen
b. proteins d. ketone bodies
Page574-576 Harper’s 26th Edition Chapter 49

116. cAMP exert its effect as a second messenger by:


a. inducing release of cytokines
b. activating Protein Kinase C
c. activating cyclic adenylase
d. activating Protein Kinase A
Page 457-468 Harper’s 26th Edition Chapter 43

117. Deficiency in the intake of carbohydrates and fats can lead to:
a. increase in protein requirements
b. increase intake of calories
c. increase deposition of fats
d. decrease in the catabolism of body proteins
Page 478-479 Harper’s 26th Edition Chapter 44

118. Gluconeogenesis is the major source blood sugar during:


a. the fed state c. prolonged fasting
b. periods between meals d. early refed state
Page 158-161 Harper’s 26th Edition Chapter 19

119. Melatonin is a specialized product of tryptophan that is involved with


a. pigment formation c. circadian rhythm
b. vasodilatation d. thyroid gland secretion
Page 267 Harper’s 26th Edition Chapter 31

120. The receptor involved in the G-protein coupled signal transduction pathway:
a. tyrosine kinase receptor c. β adrenergic receptor
b. guanylyl cyclase receptor d. janus kinase receptor
Page 459,464-465 Harper’s 26th Edition Chapter 43

121. Dinitrophenol (DNP) and other uncouplers which uncouple oxidation and phosphorylation
are useful weight losing agents. They prevent ATP sysnthesis in the Electron Transport Chain
by which of the following mechanism?
a. Electron Transport is blocked
b. Protons build up in the intermembrane space
c. There is reduced availability of ADP
d. The electrochemical gradient is dissipated
Page 95-96 Harper’s 26th Edition Chapter 12

Page 13 of 71
Page 14 of 71
Biochemistry

122. The most active form of vitamin D is:


a. 1,25 dihydroxycholecalciferol c. ergocalciferol
b. vitamin D3 d. 7- dehydrocalciferol
Page 484-486 Harper’s 26th Edition Chapter 45

123. Plant fat is high in unsaturated fatty acids compared to animal fat. Which one of the
following characteristics BEST describes unsaturated fatty acids in contrast saturated fatty
acids?
a. lower rate of oxidation
b. lower rate of melting point
c. less water soluble
d. more solid at room temperature
Page 144 Harper’s 26th Edition Chapter 14

124. This is the mechanism of action by which penicillin kills bacterial cells:
a. it inhibits bacterial DNA replication
b. it acts as an allosteric inhibitor to the enxzyme reverse transcriptase
c. it is an analogue of D-phe-D-phe effectively preventing the synthesis of
tetrahydrofolates
d. it covalently binds to the active site of the enzyme for cell wall synthesis
Page 371-372 Harper’s 26th Edition Chapter 38

125. An attack of acute asthma has been attributed to leukotrienes. Which of the following
statements is TRUE regarding leukotrienes?
a. produced mainly in the nervous tissue
b. components of slow reacting substance of anaphylaxis
c. decrease vascular permeability
d. inhibit the release of lysosomal enzymes
Page 194-196 Harper’s 26th Edition Chapter 23

126. This statement accurately describes Vitamin K:


a. Menaquinone-7 is the major form of Vitamin K found in plants
b. It is found only in green leafy vegetables while intestinal bacteria are not able to
synthesize it
c. It functions as a cofactor to a carboxylase that acts on glutamate residues of
clotting factor precursors proteins
d. It main function is to protect the erythrocytes
Page 486 Harper’s 26th Edition Chapter 45

127. Bilirubin is an important endproduct during the course of hemolysis. Which of the following
statements is TRUE regarding bilirubin?
a. excreted from the liver in the form of bilirubin diglucuronides
b. conjugation mainly it takes place in the kidneys
c. unconjugated bilirubin is readily excreted in the urine
d. unconjugated hyperbilirubinemia due to complete obstruction of the common bile
ducts is generally accompanied by increased urobilinogen in the urine
Page 280-284 Harper’s 26th Edition Chapter 32

128. Kwashiorkor occurs when there is:


a. lack of calories in the diet
b. lack of calories from protein in the diet
c. lack of carbohydrates and protein in the diet
d. lack of all the macro and micro nutrients in the diet
Page 237,478-479 Harper’s 26th Edition Chapter 28 , 44

129. Cancer can be attributed to mutation in the DNA. A mutation that converts an amino acid
codon to a stop codon is:
a. silent c. missense
b. frameshift d. nonsense
Page 362 Harper’s 26th Edition Chapter 38

130. The process by which a polypeptide’s destination in the cell is determined by which of
these?
a. Codons c. signal peptides

Page 14 of 71
Page 15 of 71
Biochemistry

b. molecular chaperones d. signal recognition particles


Page 498,503-504,508 Harper’s 26th Edition Chapter 46

131. Which of the following histones is uniquely associated with the linker DNA?
a. H3 c. H4
b. H1 d. H2A
Page 314-315 Harper’s 26th Edition Chapter 36

132. Which of the following is a nucleoside?


a. guanylic acid c. guanosine 5’-monophosphate
b. dUMP d. cytidine
Page 286-287 Harper’s 26th Edition Chapter 33

133. Which of the following about the salvage pathway of purine nucleotide synthesis is correct?
a. Deficiency of hypoxanthine-guanine phosphoribosyl transferase is the cause of
Lesch-Nyan syndrome
b. The rate limiting enzyme is PRPP amidotransferase
c. The donor of the ribose 5-phosphate is the HMP shunt
d. The salvage pathway for the free bases require more ATP than the novo sysnthesis
of purine nucleotide
Page 294 Harper’s 26th Edition Chapter 34

134. An individual who has fasted for more than 24 hours will have an increased production of
which amino acids in his tissues?
a. aspartate and glutamine
b. glutamate and asparagine
c. valine, isoleucine and leucine
d. glutamine and alanine
Page 14-20 Harper’s 26th Edition Chapter 3

135. Which amino acid is purely ketogenic ?


a. leucine c. isoleucine
b. tryptophan d. proline
Page 15,259-260 Harper’s 26th Edition Chapter 3,30

136. Kwashiorkor occurs when there is:


a. lack of calories in the diet
b. lack of calories from protein in the diet
c. lack of carbohydrates and protein in the diet
d. lack of all the macro and micro nutrients in the diet
Page 478-479 Harper’s 26th Edition Chapter 44

137. This amino acid is the source of most of the blood glucose during starvation:
a. glutamine c. glutamate
b. alanine d. aspartate
Page 236 Harper’s 26th Edition Chapter 27

138. Allopurinol is used in gout bacause it is an allosteric inhibitor of xathine oxidase which is
responsible for synthesizing this excretory metabolite of purines:
a. allanton c. urea
b. uric acid d. ammonia
Page 301 Harper’s 26th Edition Chapter 35

139. The catecholamines, epinephrine and dopamine are synthesized from this amino acid:
a. tyrosine c. arginine
b. isoleucine d. threonine
Page 445 Harper’s 26th Edition Chapter

140. Which of the following is the most important role of amino acids in the body?
a. source of energy
b. precursor of nonprotein nitrogen containing substances
c. building blocks of tissue protein
d. precursor glucose
Page 249 Harper’s 26th Edition Chapter 30

Page 15 of 71
Page 16 of 71
Biochemistry

141. Most of the ammonia that are used for the synthesis of urea are directly derived from the
deamination of:
a. leucine c. isoleucine
b. aspartic acid d. glutamic acid
Page 243-244 Harper’s 26th Edition Chapter 29

142. A person suffering from gout should decrease his intake of:
a. bread c. egg
b. banana d. sardines
Page 229 Harper’s 26th Edition Chapter 26

143. Most of the function membranes are accomplished by:


a. proteins c. carbohydrates
b. phospholipids d. cholesterol
Page 416-417 Harper’s 26th Edition Chapter 41

144. Which statements is BEST describes the arrangement of proteins and lipids in membranes?
a. Protein form a continuous sheet on either side of the phospholipid bilayer
b. Proteins occur in the lipid bilayer as discontinous particles
c. Protein molecules are sandwiched between the exoplasmic and cytoplasmic lipid
monolayers
d. Lipid molecules “float” on a sea proteins
Page 416-417 Harper’s 26th Edition Chapter 41

145. Excessive ingestion of raw egg white is detrimental because it inhibits this step in
extramitochondrial lipogenesis:
a. formation of malonyl CoA c. reduction step
b. binding of malonyl to acyl carrier protein d. deacylation reaction
Page 173-174 Harper’s 26th Edition Chapter 21

146. Gelatin has a high collagen content. It is not a good source of protein because:
a. collagen contains a limited variety of amino acids
b. collagen cannot be digested
c. too much collagen causes brittle bones
d. collagen contains more sugars than amino acids
Page 38 Harper’s 26th Edition Chapter 5

147. Semi-conservative replication of DNA means:


a. DNA systhesis occurs one strand at a time
b. Each new DNA strand forms a pair with the template strands
c. The two new DNA strands pair with each other
d. Duplication occurs in opposite direction in two strands
Page 306-307 Harper’s 26th Edition Chapter 35

148. Which of the following is the BEST describes the action of 5 FU in the treatment of cancer?
a. inhibits synthesis of DNA
b. inhibits protein synthesis
c. depletes the cancer cells of necessary nutrients like vitamins
d. depletes the cancer cells of required oxygen
Page 328-330 Harper’s 26th Edition Chapter 36

149. Which one of the following is true regarding RNA?


a. Messenger RNA contain peculiar bases
b. The codon is found in trasfer RNA
c. Ribosomal RNA contain the anticodon
d. Transfer RNA carries the amino acid
Page 306-312,341-357 Harper’s 26th Edition Chapter 35,37

150. During DNA replication, which enzyme joins Okazaki fragments?


a. topoisomerase c. DNA ligase

Page 16 of 71
Page 17 of 71
Biochemistry

b. RNAase II d. helicase
Page 328 Harper’s 26th Edition Chapter 36

151. Degeneracy of the genetic code denotes the existence of:


a. multiple codons for a single amino acid
b. codons consisting only of 2 bases
c. base triplets that do not code for any amino acid
d. codons that include one or more of the “unusual” bases
Page 359 Harper’s 26th Edition Chapter 38

152. In the normal resting state of humans, muscles will mainly use up energy provided by:
a. carbohydrates c. fats
b. branced chain amino acids d. muscle glycogen
Page 574-576 Harper’s 26th Edition Chapter 49

153. Familial hypercholesterolemia is due to


a. deficiency or defective LDL receptors
b. mutation on HMG-CoA reductase
c. increased dietary intake of cholesterol
d. decreased excretion of cholesterol
Page 228 Harper’s 26th Edition Chapter 26

154. Which of the following is the most metabolically active plasma lipid?
a. nonesterified fatty acid c. free cholesterol
b. lecithin d. glucocerebroside
Page 57 Harper’s 26th Edition Chapter 7

155. Deficiency of lipoprotein lipase will result in a increased plasma level of which of the
following?
a. chylomicron remnant c. LDL
b. VLDL d. HDL
Page 125,126,207-208 Harper’s 26th Edition Chapter 15,25

156. Which of the following is needed for the transport of long-chain fatty acids from the cytosol
to the mitochondrial matrix?
a. carnitine c. glutathione
b. S-adenosylmethionine d. phosphoadenosine phosphosulfate
Page 180-187 Harper’s 26th Edition Chapter 22

157. Major enzyme that digest dietary triacylglycerol


a. lingual lipase c. lipoprotein lipase
b. pancreatic lipase d. hormone sensitive lipase
Page 475-476 Harper’s 26th Edition Chapter 44
158. In the liver cholesterol is mainly disposed by:
a. being exported as HDL
b. being exported as VLDL
c. being secreted in the bile as free cholesterol
d. being converted to bile acids
Page 225-227 Harper’s 26th Edition Chapter 26

159. Which is the immediate source of energy of ATP synthesis?


a. electron transfer to O2 c. ADP and Pi
b. electrochemical gradient d. solar energy
Page 96-97 Harper’s 26th Edition Chapter 12

160. The link between the catabolic and anabolic reactions in the energy cycle is:
a. ATP c. H+
b. O2 d. NADH
Page 82-85 Harper’s 26th Edition Chapter 10

161. Which of the following is a function of pyridoxal phosphate?


a. prosthetic group of transaminases
b. lowers plasma cholesterol
c. a coenzyme to many dehydrogenases

Page 17 of 71
Page 18 of 71
Biochemistry

d. adds hydride ion to the pyridine ring


Page 50,491 Harper’s 26th Edition Chapter 7,45

162. What is the role of oxygen in cellular respiration?


a. source of electrons to be passed along the respiratory chain
b. final receptor of electrons in the respiratory chain
c. transporter of electrons from cytosol to mitochondrial matrix
d. cofactor of ATP synthase
Page 92-101 Harper’s 26th Edition Chapter 12

163. Which vitamin is solely synthesized by bacteria and is thus absent from all plants?
a. Biotin c. Folic acid
b. Ascorbic acid d. Riboflavin
Page 494-495 Harper’s 26th Edition Chapter 45

164. This vitamin is required for the synthesis of neurotransmitters like epinephrine:
a. Niacin c. Pyridoxine
b. Ascorbic acid d. Riboflavin
Page495-496 Harper’s 26th Edition Chapter 45

165. Deficiency of this mineral is prevalent among pregnant women living in mountainous areas:
a. iron c. copper
b. iodine d. magnesium
Page 447-449 Harper’s 26th Edition Chapter 42

166. Which of the ff. accumulates in carnitine deficiency in humans?


A. glycogen
B. odd-numbered fatty acids *
C. triglycerides
D. sphingolipids

167. Which regulatory action involves a reversible covalent modification of an enzyme?


A. allosteric modulation
B. competitive inhibition
C. association of apoenzyme with a cofactor
D. phosphorylation of Ser hydroxyl on the enzyme *

168. The Ping-pong mechanism of enzyme kinetics applies to which of the reactions?
A. one-substrate
B. bisubstrate *
C. multiple substrate
D. hydrolytic

169. Enzymes hasten the rate of chemical reactions through which of the ff. ?
A. covalent modification of coenzymes and reactants
B. enhancing proximity and local concentration of substrates
C. decreasing the free energy of activation *
D. increasing the over-all free energy change of the reaction

170. What is the role of oxygen in electron transport system ?


A. accepts electrons from FADH2
B. the final acceptor of electrons *
C. a mobile electron carrier
D. donates electron to water

171. Which of the ff. BEST explains why oxidation of FADH2 produces less ATP than NADH?
A. FADH2 enters ETS via complex II *
B. FADH2 transfers less electrons through ETS than NADH
C. Electrons from FADH2 are not transferred to oxygen
D. inner mitochondrial membrane is impermeable to FADH2

172. Which of the ff. BEST applies to Isoelectric ph of a protein?


A. ph is physiologic
B. net charge is zero *

Page 18 of 71
Page 19 of 71
Biochemistry

C. protonation is maximum
D. ionization is maximum

173. What kind of a reaction is interconversion of aldose and ketose ?


A. dehydration
B. reduction
C. oxidation
D. isomerization *

174. In the over-all scheme of human metabolism, which organ plays impt role?
A. brain
B. liver *
C. muscle
D. kidney

175. Which of these enzymes is important in protein digestion ?


A. ptyalin
B. trypsin *
C. invertase
D. pancreatic amylase

176. The beta pleated sheet of a protein is classified as which level of protein ?
A. primary C. tertiary
B. secondary * D. super secondary

177. What is the cause for the lactic acidosis that develops in thiamine deficiency?
A. conversion of pyruvate to acetyl-CoA is blocked *
B. conversion of pyruvate to oxaloacetate is blocked
C. the transketolase reaction that transfers 3-carbon units is blocked
D. the transaldolase reaction has increased activity

178. Which of the ff. statements BEST describes collagen?


A. left handed helix
B. 2 left handed alpha helices
C. double helical structure
D. triple helix *

179. Which of these BEST explains the role of HCL in the stomach?
A. kills H. pylori organisms
B. denatures proteins *
C. ideal ph for amylase action
D. stimulates pancreatic juice sec’n

180. Which of the ff. bonds stabilizes the primary structure of a protein?
A. hydrogen *
B. disulfide
C. ionic
D. peptide

181. Which of the ff. are products of the action of salivary amylase on glycogen?
A. sucrose
B. maltose *
C. galactose
D. fructose

182. A zwiterion is a molecule that contains which of the following?


A. a cation and anion * C. polar and non-polar groups
B. more than one cation D. more than one anion

183. Which of the ff. is most important source of glucose during early fasting?
A. liver triglyceride C. muscle protein
B. hepatic glycogen * D. muscle glycogen

Page 19 of 71
Page 20 of 71
Biochemistry

184. A young infant, who was nourished by synthetic formula, had sugar in blood & urine. This
compound gave positive reducing sugar test but negative with glucose oxidase test. Which
of the ff. compounds is MOST likely to be present in infant’s blood & urine?
A. glucose C. Sorbitol *
B. fructose D. maltose

185. In normal resting state of humans, muscles will mainly use up energy that is provided by
which of the ff.?
A. carbohydrates C. fats *
B. muscle glycogen D. amino acids

186. The anemia that develops in patients on anti-malarial therapy may be due to deficiency of
enzymes that belong to which metabolic pathway?
A. Embden-Mayerhoff C. hexose monophosphate *
B. Kreb’s TCA D. Kreb’s-Hanseleit

187. Which of the ff is the primary function of the Pentose Phosphate pathway?
A. ATP production
B. NADPH generation
C. NADPH generation & production of ribose units *
D. Provision of ribose units for nucleotide/ nucleic acid synthesis

188. Which of the ff, is the major metabolic substrate required by RBC?
A. Glycerol C. fructose

B. free fatty acids D. glucose *

189. Which of the ff. is true of the TATA box?


A. binds to anti-codon C. encodes repressor proteins
B. binds RNA polymerase * D. located in first structural gene

190. SRS-A, the slow reacting substance of anaphylaxis, is composed of which of the ff?
A. thromboxanes C. leukotrienes *
B. prostaglandins D. prostacyclins

191. The regulation of cholesterol synthesis primarily occurs at which enzymatic site?
A. 3-hydroxy-3-methylglutaryl CoA synthase
B.3-hydroxy-3-methylglutaryl CoA lyase
C.3-hydroxy-3-methylglutaryl CoA reductase *
D.3-methyl-3-hydroxylglutaryl CoA oxidase

192. What is the major source of reducing equivalents used for fatty acid syn?
A. pentose phosphate pathway *
B. Embden-Mayerhoff pathway
C. the tricarboxylic acid cycle
D. Pyruvate dehydrogenase pathway

193. What is the role of aspirin in the treatment of rheumatoid arthritis?


A. inhibits the release of arachidonic acid
B. directly neutralizes prostaglandins & thromboxanes
C. inactivates cyclooxygenase *
D. shuts down the synthesis of all eicosanoid

194. Patients suffering from familial defect in Apo B 100 have significantly elevated levels of
plasma LDL’s. Which is the major reason for this?
A. inability of LDL to interact with the LDL receptor *
B. inability of LDL to activate lipoprotein lipase
C. inability of LDL to activate cholesterol transfer protein
D. LDL interaction with LDL receptors fail to stimulate endocytosis

195. What is the pharmacological benefit of the drug – Lovastatin ?

Page 20 of 71
Page 21 of 71
Biochemistry

A. inhibits intestinal absorption of dietary cholesterol


B. inhibits hepatic production of VLDL
C. inhibits HMG-CoA reductase *
D. absorbs bile acids, increasing their rate of elimination in the feces

196. In an average adult, which % of body protein is turned over on a daily basis?
A. 1 – 2 % * C. 5 – 6
B. 3 – 4 % D. 7 – 8 %

197. Which vitamin combines with opsin to form the visual pigment in the eye ?
A. A * C. E
B. D D. K

198. Which of the following is the cause of muscle fatigue following vigorous exercise?
A. sodium and potassium imbalance
B. the utilization of ATP
C. accumulation of ADP and PI
D. accumulation of lactate *

199. Which intermediate of pyrimidine biosynthesis is common to the urea cycle?


A. ornithine C. carbamoyl phosphate *
B. citrulline D. carbamoyl aspartate

200. Which of these is a function of nonsense codons?


A. they code for non-essential amino acids
B. they are responsible for degeneracy of the genetic code
C. they base pair with the anticodons
D. they may be used in the cell as termination signals *

201. Which phase of protein synthesis does the release of newly synthesized polypeptide from
the ribosome take place?
A. initiation C. release
B. elongation D. termination *

202. Which of the ff. directs the polypeptide’s ultimate destination in the cell ?
A. codons C. signal peptides *
B. molecular chaperones D. signal recognition particles

203. What is the primary action of methotrexate, a chemotherapeutic agent ?


A. displaces cells of folate * C. inhibits phosphorylation of free bases
B. inhibits cellular kinase activity D. inhibits synthesis of nucleotides

204. Which of these is the first reaction in the metabolism of all sugars ?
A. hydrolysis C. isomerization
B. proteolysis D. phosphorylation *
205. Which site on the operon is the site of RNA polymerase binding ?
A. operator C. promoter *
B. regulator D. structural

206. What is the cellular site of synthesis of messenger RNA ?


A. nucleus C. mitochondria
B. cytosol D. Ribosome *

207. Which of the following BEST applies to transfer RNA ?


A. largest of RNA specie C. exists in at least 20 diff forms *
B. has no secondary structure D. assoc. w/ histones in cytosol

208. Which of the ff. is the biological function of DNA synthesis ?


A. production of normal RNA’s C. prevents mutation
B. production of normal proteins D. assures viability of the specie *

209. During RNA synthesis, DNA template sequence 5’ TpApGpCp 3’ would be transcribed to
produce which of the ff. sequences ?

Page 21 of 71
Page 22 of 71
Biochemistry

A. 5’- TpApCpGp- 3’ C. 3’- ApUpCpGp-5’


B. 3’- GpCpTpAp- 5’ D. 5’- GpCpUpAp-3’ *

210. Which of the ff. refer to sequential order of nucleotides in a polypeptide?


A. primary structure * C. tertiary structure
B. secondary structure D. quaternary structure

211. Which of the ff. enzymes in replication functions as proof-reading device?


A. polymerase enzyme C. 5’ exonuclease
B. 3’ exonuclease * D. nuclease

212. Which of these constitute the second level of organization of DNA?


A. nucleosome * C. double helix DNA
B. chromosome D. condensed metaphase chromosome

213. Which of the ff. BEST describes Okazaki fragments?


A. fragments of DNA attached to an initiator component
B. fragments of continuous DNA which eventually are joined to form DNA
C. fragments of DNA that are copied in direction away from replic’n fork *
D. synthesized fr. 5’ to 3’ & proceeds in same direction as replication fork

214. The antimetabolite, 5-FU inhibits DNA replication by which mechanism?


A. intercalates into the DNA
B. cross-links adjacent thymines in the DNA
C. inhibts production of deoxythymidine monophosphate *
D. cross-links adjacent guanine residues in the DNA

215. Which posttranscriptional processing prevents 5’ exonuclease attack?


A. splicing reactions
B. B.addition of extra COOH groups
C. poly A tail *
D. 7 methylguanosine triPO4

216. Which of the ff. reactions does RNA-directed DNA polymerase catalyze?
A. synthesis of DNA complimentary to the RNA strand *
B.synthesis of DNA complimentary to template DNA strand
C. removal of wrong nucleotide inserted during DNA synthesis
D.synthesis of an RNA that is complimentary to RNA template

217. During protein synthesis, which reads information in mRNA & transfers AA to a growing
polypeptide chain?
A. messenger RNA C. heterogenous nuclear RNA
B. ribosomal RNA D. transfer RNA *

218. What is the cause of polyuria in diabetes mellitus ?


A. overdrinking of water because of polydipsia
B. decreased ADH activity
C. osmotic diuresis *
D. decreased aldosterone activity

219. Which is TRUE about methemoglobinemias?


A. There is increased activity of methemoglobin reductase.
B. It can result as a side-effect of sulfonamides.
C. Increased reduction of Fe3+ of methemoglobin to Fe2+ occurs.
D. There is absence of the Bohr effect.
Reference: Murray, R.K., et al., Harper’s Biochemistry, 27 th Ed., pp. 46-47.

220. One of the following is a saturated fatty acid:


A. Oleic acid
B B. Linolenic acid
C C. Palmitic acid
D. Arachidonic acid
Reference: Murray, R.K., et al., Harper’s Biochemistry, 27 th Ed., p. 123.

Page 22 of 71
Page 23 of 71
Biochemistry

221. One of the following statements is true of the structure


CH3(CH2)4CH=CHCH2CH=CH(CH2)7COOH:
A. It is a saturated fatty acid with 18 C atoms.
B. Its common name is linolenic acid.
C. It has two double bonds located at C 9 and C 12.
D. Its other name is 18:3c9,12.
Reference: Murray, R.K., et al., Harper’s Biochemistry, 27 th Ed., p. 122.

Lipid Metabolism
222. Which of the following is TRUE of fatty acid synthase enzyme complex?
A. It can synthesize two 18 carbon atom fatty acids simultaneously.
B. It is composed of two polypeptide chains but each monomer is an active enzyme.
C. FAS enzyme complex has seven sub-units with catalytic activities.
D. The monomers are arranged in a head – to – tail configuration.
Reference: Murray, R.K., et al., Harper’s Biochemistry, 27 th Ed., pp. 196-197.

223. The phosphopanthetheine side chain is part of which sub-unit of the fatty acid synthase
enzyme complex ?
A. acetyl transacylase C. acyl carrier protein
B. ketoacyl synthase D. malonyl transcylase
Reference: Murray, R.K., et al., Harper’s Biochemistry, 27 th Ed., p. 196.

224. Deficiency of this phospholipid results to Respiratory Distress Syndrome of the newborn:
A. phosphatidylinositol C. phosphatidylserine
B. phosphatidylinositide D. dipalmitoyl-lecithin
Reference: Murray, R.K., et al., Harper’s Biochemistry, 27 th Ed., p. 125.

225. The committed step for the β – oxidation of fatty acids is inhibited by increased
concentration of:
A. acetyl-CoA C. malonyl-CoA
B. carnitine D. palmitoyl – CoA
Reference: Murray, R.K., et al., Harper’s Biochemistry, 27 th Ed., p. 195.

226. The complete ATP yield of a saturated C16 fatty acid is:
A. 106 C. 110
B. 108 D. 112
Reference: Murray, R.K., et al., Harper’s Biochemistry, 27 th Ed., p.189.

227. Which condition accelerates ketogenesis?


A. high carbohydrate diet
B. Increased [NAD+]/[NADH] ratio
C. low [beta-hydroxybutyrate]
D. decreased [Insulin]/[glucagon] ratio
Reference: Murray, R.K., et al., Harper’s Biochemistry, 27 th Ed., pp. 192-193.

228. Ketone bodies can be utilized by extra-hepatic tissues because of this enzyme:
A. beta-hydroxybutyrate dehydrogenase C. HMG-CoA lyase
B. HMG-CoA synthase D. succinyl CoA acetoacetate CoA
transferase
Reference: Murray, R.K., et al., Harper’s Biochemistry, 27 th Ed., p. 191.

229. Accelerated ketogenesis occurs in which situation?


A. activation of CPT I C.  glucagon/Insulin
B.  malonyl CoA D. activation of acetyl CoA carboxylase

Murray, R.K., et. al., Harper's Illustrated Biochemistry, 27 th ed., p.193

230. This disease results from the accumulation of phytanic acid in the brain and kidneys of
patients:
A. Tay- Sach’s disease C. Refsum’s disease
B. Zellweger’s disease D. Niemann Pick’s disease
Reference: Murray, R.K., et al., Harper’s Biochemistry, 27 th Ed., p. 194.

231. Which is TRUE regarding apoproteins?

Page 23 of 71
Page 24 of 71
Biochemistry

A. Apo B 46 and B100 are synthesized in the liver.


B. Apo A, B, C and D are peripheral apoproteins.
C. Apo D is present exclusively in HDL.
D. Apo A II and C III are stimulate endothelial lipoprotein lipase.
Reference: Murray, R.K., et al., Harper’s Biochemistry, 27 th Ed., pp. 218.

232. The concentration of this lipoprotein is inversely related to the incidence of coronary artery
disease:
A. HDL1 C. HDL3
B. HDL2 D. pre β- HDL
Reference: Murray, R.K., et al., Harper’s Biochemistry, 27 th Ed., p. 222.

233. Which receptors are actively involved in reverse cholesterol transport?


A. LDL C. LRP
B. ABCA 1 D. SR- B1
Reference: Murray, R.K., et al., Harper’s Biochemistry, 27 th Ed., p. 222.

234. The only part of an odd-chain fatty acid that is glucogenic is:
A. acetyl CoA C. propionyl CoA
B. butyryl CoA D. succinyl CoA
Reference: Murray, R.K., et. al., Harper's Illustrated Biochemistry, 27 th ed., p.188

235. Which is the correct disease: enzyme deficiency pair?


A. Fabry's disease: Ceramidase
B. Gaucher's disease: Hexosaminidase A
C. Niemann - Pick disease: Sphingomyelinase
D. Tay-Sachs disease:  - glucosidase
Reference: Murray, R.K., et. al., Harper's Illustrated Biochemistry, 27 th ed., p. 215

236. A patient with phenylketonuria has fair skin and eyes due to low levels of:
A. biopterine C. tryptophan
B. phenylalanine D. tyrosine
Reference: Murray, R.K., et al., Harper’s Biochemistry, 27 th Ed., p. 242.

237. The preferred amino acid as substrate for gluconeogensis is:


A. alanine C. asparagine
B. arginine D. aspartate
Reference: Murray, R.K., et al., Harper’s Biochemistry, 27 th Ed., p. 173 .

238. A patient with mental retardation and pigmentation of connective tissues has a deficiency
of:
A. alanine aminotransferase C. homogentisate oxidase
B. aspartate aminotransferase D. phenylalanine hydroxylase
Reference: Murray, R.K., et al., Harper’s Biochemistry, 27 th Ed., p. 259.

For questions 238-239, refer to the case below.


D.D., a 2 week old male infant has failure to gain weight, nausea and vomiting after breast feeding.
Blood examination revealed increased NH3+.

239. If D.D. had ↑ blood level of argininosuccinate and friable hair, the deficient enzyme is:
A. Argininosuccinate synthethase C. Carbamoyl P Synthetase 1
B. Argininosuccinate lyase D. Ornithine transcarbamoylase
Reference: Murray, R.K., et al., Harper’s Biochemistry, 27 th Ed., p. 252.

240. The immediate treatment for D.D. is:


A. antibiotics administration C. protein restriction
B. hippurate administration D. IV fluids
Reference: Murray, R.K., et al., Harper’s Biochemistry, 27 th Ed., p. 251.

For questions 241-242, refer to the reaction below.


NH3+ O
│ ║
R1- C – COO- R 1- C – COO-

Page 24 of 71
Page 25 of 71
Biochemistry

NH3+
O │
║ R2- C – COO- R 2- C
– COO-

241. This reaction requires which coenzyme?


A. ATP C. NAD+
B. FAD +
D. Vit. B6
Reference: Murray, R.K., et al., Harper’s Biochemistry, 27 th Ed., p. 247.

242. The final products of this reaction are:


A. alanine and pyruvate
B. glutamate and α-ketoglutarate
C. glutamate and oxaloacetate
D. glutamine and oxaloacetate
Reference: Murray, R.K., et al., Harper’s Biochemistry, 27 th Ed., p. 248.

243. The reaction that helps maintain normal acid-base balance is:
A. arginase C. glutamate dehydrogenase
B. glutaminase D. glutamine synthetase
Reference: Murray, R.K., et al., Harper’s Biochemistry, 27 th Ed., p. 249.

244. GABA is an inhibitory neurotransmitter because it causes:


A. ↑ Cl- efflux C. ↑ Cl- influx
B. ↓↓
Ca++ efflux D. ↓ K+ influx
Reference: Murray, R.K., et al., Harper’s Biochemistry, 27 th Ed., p. 274.

245. Which amino acid test dose administration is used to diagnose folate deficiency?
A. cysteine C. lysine
B. histidine D. tryptophan
th
Reference: Murray, R.K., et al., Harper’s Biochemistry, 27 Ed., p. 254.

246. Which amino acid test dose administration is used to diagnose pyridoxal phosphate
deficiency?
A. histidine C. tryptophan
B. leucine D. tyrosine
Reference: Murray, R.K., et al., Harper’s Biochemistry, 27 th Ed., p. 262.

247. To prevent obtaining a false-positive test for phenylketonuria, the screening should be done
how many days after birth?
A. immediately after birth C. 2 days
B. one day D. 3 - 4 days
Reference: Murray, R.K., et al., Harper’s Biochemistry, 27 th Ed., p. 259.

248. A 10 year old boy with pellagra-like symptoms and a urine finding of ↑ amount of large
neutral amino acids (aminoaciduria). What is the problem with this patient?
A. homocystinuria C. glycinuria
B. cystinuria D. Hartnup
Reference: Murray, R.K., et al., Harper’s Biochemistry, 27 th Ed., p. 262.

249. One of the following statements about the THYROGLOBULIN is TRUE:


A. It is the precursor of calcitonin
B. It is iodinated, glycosylated carbohydrate with a molecular mass of 660 kilodalton
C. It contains 115 tyrosine residues.
D. It is initially synthesized in the apical portion of the cell
Reference: Murray, R.K., et al., Harper’s Biochemistry, 27 th Ed., p. 455.

250. One of the following events is TRUE of the concentration of IODIDE:


A. It is an energy dependent process and is linked to the ATPase dependent Na-K pump.
B. Thyroperoxidase is essential enzyme for the concentration of Iodide.
C. It involves the iodination of the 3 position of the aromatic ring
D. Phagocytosis and pinocytosis are essential in this process

Page 25 of 71
Page 26 of 71
Biochemistry

Reference: Murray, R.K., et al., Harper’s Biochemistry, 27 th Ed., p. 456.


251. The half-life of T4 in blood is:
A. 1.5 days C. 4.5 days
B. 3.5 days D. 6.5 days
Reference: Murray, R.K., et al., Harper’s Biochemistry, 27 th Ed., p. 462.

252. One of the following statements about THYROID BINDING GLOBULIN is TRUE:
A. Most of the T4 and T3 is in the unbound form.
B. Thyroxine Binding prealbuminis quantitatively more important.
C. Thyroxine binding globulin binds covalently nearly all thye T3 and T4
D. Estrogens increases the synthesis of thyroid Binding globulin.
Reference: Murray, R.K., et al., Harper’s Biochemistry, 27 th Ed., p. 462.

253. After exposing a jaundiced infant to phototherapy, the following is TRUE of bilirubin
excretion:
A. Bilirubin is converted to bilirubin diglucoronide.
B. Phototherapy provides the sulfates needed to conjugate bilirubin.
C. Bilirubin will be excreted in the feces as mesobilirubin.
D. Increased amounts of urobilin are found in the urine.
Reference: Murray, R.K., et. al., Harper's Illustrated Biochemistry, 26 th ed., p. 280

Blood Coagulation
254. In normal patients, the fibrinolytic system of the coagulation system is have the following
characteristized by:
A. stabilization of the fibrin molecules by covalent cross-linkage bonding.
B. glycoproteins that regulate coagulation mechanism.
C. components which are thiol-dependent transglutaminases.
D. glycoproteins that contain serine dependent active protease sites.
Reference: Murray, R.K., et. al., Harper's Illustrated Biochemistry, 26 th ed., p. 604

255. Intracellular calcium is mobilized in the cytoplasm by this PIP 2-derived second messenger.
A. Protein kinase C C. Inositol diphosphate
B. Inositol triphosphate D. Phosphatidylinositol
Reference: Murray, R.K., et. al., Harper's Illustrated Biochemistry, 25 th ed., p. 546

256. During the follicular phase of the menstrual cycle:


A. there is a sudden increase of estradiol as the follicle enlarges
B. there is proliferation of the endometrial lining
C. the sudden drop in the levels of FSH heralds the end of follicular phase
D. All of the above are TRUE
Reference: Murray, R.K., et. al., Harper's Illustrated Biochemistry, 23rd Ed., p. 550

257. Which of the following statement/s is/are TRUE of estrogen?


A. In, pregnancy, more estrone is coming from the placenta
B. Estrogens are formed by the aromatization of androgens in a complex process that
involves 2 hydroxylation steps
C. As much as 50% of estradiol produced during pregnancy are produced by the ovaries
D. All the statements are TRUE
Reference: Murray, R.K., et. al., Harper's Illustrated Biochemistry, 23rd ed., p. 450

258. The following is TRUE about the luteal Phase of the menstrual cycle.
A. There is follicular development
B. The endometrial lining is thickened with the glands.
C. There is low levels of estrogen.
D. FSH and LH levels are high

259. Which statement/s is/are TRUE about mineralocorticoids?


A. They are secreted from the zona fasciculata of the adrenal cortex
B. The final step in the synthesis is conversion of an aldehyde to an alcohol
C. Synthesis of mineralocorticoids depends on the presence of the enzyme 18- hydroxylase
D. Mineralocorticoid comes from the parent compound Pregnenolone.
Reference: Murray, R.K., et. al., Harper's Illustrated Biochemistry, 27 th ed., p. 446.

260. Which of the following is TRUE regarding the glycemic index?

Page 26 of 71
Page 27 of 71
Biochemistry

A. Foods with a low glycemic index cause less fluctuations in insulin secretion.
B. Foods with a high glycemic index cause rapid fluctuations in insulin secretion
C. Glucose and galactose have lower glycemic index than non-starch polysaccharides.
D. Fructose and sugar alcohols are absorbed rapidly and have a higher glycemic
index.
Reference: Murray, R.K., et al., Harper’s Biochemistry, 27 th Ed., p. 485.

261. This water-soluble vitamin requires a specific transport protein:


A. Cobalamin C. Folic acid
B. Thiamine D. Riboflavin
Reference: Murray, R.K., et al., Harper’s Biochemistry, 27 th Ed., p. 486.

262. Which of the following statements about factors affecting the basal metabolic rate is TRUE?
A. The basal metabolic rate is higher in females due to greater adiposity
B. Increase in body weight decreases the basal metabolic rate
C. Basal metabolic rate decreases with advancing age
D. Greater muscle tissue means a decrease in basal metabolic rate
Reference: Murray, R.K., et al., Harper’s Biochemistry, 27 th Ed., p. 491.

263. The form of Vitamin A involved in regulation of gene expression is:


A. Retinol C. Retinyl phosphate
B. Retinaldehyde D. Retinoic acid
Reference: Murray, R.K., et al., Harper’s Biochemistry, 27 th Ed., p. 492.

264. The main storage form of Vitamin D is:


A. 7-dehydrocholesterol C. 25-hydroxycholecalciferol
B. Cholecalciferol D. 1,25 –dihydroxycholecalciferol
Reference: Murray, R.K., et al., Harper’s Biochemistry, 27 th Ed., p. 500.

265. This vitamin participates in the transfer of 1-carbon methyl units:


A. Folic acid C. Riboflavin
B. Pantothenic acid D. Thiamine
Reference: Murray, R.K., et al., Harper’s Biochemistry, 27 th Ed., p. 500.

266. The following peptides arise from the proopiomelanocortin gene except:
A. Melanocyte-stimulating hormone
B. Corticotropin-like intermediate lobe peptide
C. Adrenocorticotrophic hormone
D. Gonadotropin releasing hormone
Reference: Murray, R.K., et al., Harper’s Biochemistry, 27 th Ed., p. 459.

267. Which of the following hormones is NOT a glycoprotein?


A. Thyroid stimulating hormone
B. Prolactin
C. Follicle stimulating hormone
D. Human chorionic gonadotropin
Reference: Murray, R.K., et al., Harper’s Biochemistry, 27 th Ed., p. 447.

268. Which of the following is the most utilized pathway of testosterone synthesis in the human
testes?
A. Dehydroepiandrosterone pathway
B. Progesterone pathway
C. Aromatase pathway
D. 21-hydroxylase pathway
Reference: Murray, R.K., et al., Harper’s Biochemistry, 27 th Ed., p. 450.

269. Conversion of cholesterol to pregnenolone in the mitochondria of the testes is promoted


by:
A. Aromatase enzyme
B. Follicle-stimulating hormone
C. Adrenocorticotrophic hormone
D. Luteinizing hormone
Reference: Murray, R.K., et al., Harper’s Biochemistry, 27 th Ed., p. 450.

Page 27 of 71
Page 28 of 71
Biochemistry

270. Monosaccharides are characterized as:


A. Carbohydrates that can be hydrolyzed to simpler carbohydrates.
B. Are classified based on the number of carbon atoms.
C. Are condensation products of simpler units.
D. Occur in linear configuration.
Reference: Murray, R.K., et al., Harper’s Biochemistry, 27 th Ed., Page 112

271. The glycemic index is a measure of:


A. polarity C. solubility
B. digestibility D. amphotericity
Reference: Murray, R.K., et al., Harper’s Biochemistry, 27 th Ed., Page 116

272. The homopolymer structure of starch is called:


A. dextrin C. glucosan
B. amylopectin D. amylose
Reference: Murray, R.K., et al., Harper’s Biochemistry, 27 th Ed., Page 116

273. The enzymic hydrolysis of starch by amylase produces:


A. lactose C. lactulose
B. maltose D. sucrose
Page 112
274. The phosphorylation of glucose during glycolysis involves:
A. an enzyme located in the mitochondrion
B. GTP as the ATP donor
C. A reversible reaction.
D. an enzyme inhibited allosterically by the product
Page 151

275. Which of the following is NOT a coenzyme of the pyruvate dehydrogenase complex?
A. biotin C. coenzyme A
B. lipoic acid D. pyridoxine
Page 155

276. This enzyme of the Krebs Cycle catalyzes a substrate level phosphorylation reaction:
A. isocitrate dehydrogenase C. citrate synthase
B. malate dehydrogenase D. succinate thiokinase
Page 146
277. Which is not a reaction that synthesizes purine nucleotides?
A. synthesis form amphibolic intermediates
B. phosphoribosylation of nucleobases
C. ribosylation of purine nucleosides
D. phosphorylation of purine nucleosides
Page 301

278. The multifunctional enzyme involved in the rate limiting reaction during pyrimidine
synthesis includes:
A. IMP dehydrogenase C. orotidylic decarboxylase
B. dihydroorotase D. orotate phosphoribosyl transferase
Page 307

279. Gout is a metabolic disorder caused by decreased activity of:


A. hypoxanthine-guanine phosphoribosyltransferase
B. adenosine deaminase
C. glucose 6 – phosphatase
D. xanthine oxidase
Page 308

280. The endproducts of pyrimidine catabolism are:


A. less water soluble than those of purine catabolism
B. increased in patients with leukemia
C. produced by adenosine deaminase
D. increased with increased dietary intake of nucleotides
Page 308

Page 28 of 71
Page 29 of 71
Biochemistry

281. AcetylCoA derived from glycolysis is:


A. reduced to water in the TCAC
B. converted to NADPH in the hexose monophosphate shunt
C. oxidized to CO2 and H2O in the TCAC
D. converted by pyruvate dehydrogenase to form lactate
Page 134

282. Which metabolic event occurs during the fed state?


A. There is depletion of metabolic fuels.
B. There is insulin-mediated uptake of glucose in the liver.
C. Extracellular lipoprotein lipase is activated in reponse to insulin.
D. Increased secretion of glucagons by the pancreas.
Page 140

283. Muscle glycogen is not exported for utilization by peripheral tissues because of the absence
of:
A. gluokinase C. phosphofructokinse
B. glycogen phosphorylaseD. glucose 6 – phosphatase
Page 142

284. Which metabolic fuel provides the energy during prolonged starvation?
A. glucose C. amino acids
B. ketone bodies D. lactate
Page 143

285. The metabolic pathway that provides the fuel for erythrocytes is:
A. β-oxidation C. anaerobic glycoslysis
B. Tricarboxylci acid cycle D. ketogenesis
Page 143

286. Which statement best describes RNA?


A. The sugar moiety attached to the base is deoxyribose.
B. The guanine content is equal to the cytosine content.
C. RNA contains ribonucloetide uracil.
D. RNA cannot be hydrolyzed by alkali.
Page 314

287. The adapter molecule in translation is:


A. mRNA C. rRNA
B. tRNA D. snRNA
Page 318

288. The human telomere sequence is described as:


A. consisting of short repeat T-G rich sequences
B. looped domains attached to the 3’ end of the chromosome
C. binds sister chromatids during spindle formation
D. involved with the aging when elongated
Page 326

289. Which of the following features is NOT true of mitochondrial DNA?


A. Encodes the 13 proteins involved in the respiratory chain
B. AGA and AGG are stop codons.
C. Contains 200,000 base pairs.
D. Is circular and double stranded.
Page 331

290. The synthesis of the forward strand during replication is described as:
A. a continuous process moving from the 3’ to 5’ direction.
B. producing short DNA fragments
C. catalyzed by DNA polymerase  in humans
D. requiring an RNA primer for each short strand formed
Page 336

Page 29 of 71
Page 30 of 71
Biochemistry

291. Xeroderma pigmentosa results from ultraviolet damage to:


A. the ORIC C. TG rich palindromes
B. thymine dimers D. GATC sequence
Page 345

292. The DNA strand that is transcribed is referred to as the:


A. coding strand C. template strand
B. nontemplate strand D. transcript strand
Page 348

293. This RNA polymerase II is responsible for the synthesis of:


A. rRNA C. tRNA
B. mRNA D. snRNA
Page 351

294. The rate of transcription may be enhanced by the following sequence elements, except:
A. TATA box C. enhancers
B. sigma factor D. silencers
Page 355

295. In eukaryotes, the sequence that is translated to proteins is called:


A. exons C. introns
B. cistrons D. spliceosomes
Page 359

296. The enzyme that catalyzes the activation of amino acids is:
A. peptidyl transferase C. aminoacyl-tRNA synthetase
B. RNA polymerase D. methylguanosine synthase
Page 367
297. Frameshift mutation results from:
A. deletion of a triplet C. insertion of a nucleotide
B. insertion of an amino acid D. deletion of an amino acid
Page 370
298. The role of polyA tail during translation is observed during:
A. elongation C. Termination
B. initiation D. modification
Page 372

299. Which antibiotic prevents binding of aminoacyl-tRNA to the A site?


A. chloramphenicol C. tetracyclin
B. puromycin D. rifampicin
Page 378

300. DNA library that is composed of DNA copies from mRNA populations:
A. chimeric library C. cDNA
B. Colony D. genomic library
Page 408

301. Insulin promotes glucose storage in the liver and muscles by:
A. inhibition of PFK 2.
B. stimulation of G-6-DH.
C. activation of glycogen synthase.
D. increased synthesis of F-2,6 bisP.
Reference: Murray, R.K., et al., Harper’s Biochemistry, 27 th Ed., p. 474.

302. A patient's DNA was analyzed for its chemical characteristics. It was found to have a high
melting temperature which means that the DNA
A. is rich in adenine-thymine base pairs
B. is denatured at a higher temperature
C. will exhibit hypochromicity at this temperature
D. cannot undergo renaturation
Harper's Biochemistry 27th edition, pp. 312-313 Application

Page 30 of 71
Page 31 of 71
Biochemistry

303. A 20 year old male suffering from pneumonia was given a quinolone antibiotics by his
physician. This antibiotics inhibits bacterial DNA topoisomerase. Which step in replication is
inhibited by the quinolone?
A. Unwinding of the double helix.
B. Recognition of the origin of replication
C. Formation of the RNA primer
D. Elongation of the growing DNA polymer
Harper's 27th edition, pp. 339 - 340 Application

304. A dermatologist advised her patient to avoid too much exposure to sunlight. Aside from
sunburn, ultraviolet light causes what kind of damage to DNA?
A. Insertion or deletion of nucleotide
B. Oxidative free radical formation
C. Formation of thymine-thymine dimer
D. Cross-linkage between bases
Harper's 27th edition, p. 343 Application

305. Cells cultured from patients with xeroderma pigmentosum exhibit low activity to which kind
of DNA repair mechanism?
A. Nucleotide excision-repair.
B. Base excision repair.
C. Mismatch repair
D. double-strand break repair
Harper's 27th edition, p. 345 Recall

306. Damage to the mitochondria may result in impaired mitochondrial DNA synthesis. In
mammalian cells, this DNA is synthesized by which type of DNA polymerase?
A. DNA polymerase alpha C. DNA polymerase delta
B. DNA polymerase beta D. DNA polymerase gamma
Harper's 27th edition, p. 336 Recall

307. Mammalian RNA polymerase II exhibit high sensitivity to the inhibitory effect of alpha-
amanitin, a peptide toxin from a poisonous mushroom. Which of the following products will
be decreased?
A. ribosomal RNA C. messenger RNA
B. transfer RNA D. small nuclear RNA
Harper's 27th edition, p. 351 Application

308. Rifampicin, an anti-TB drug, inhibits DNA-dependent RNA polymerase. Which of the
following steps of gene transcription will NOT be affected by the drug?
A. Template binding C. Chain initiation
B. Chain elongation D. Posttranscriptional modification
Harper's 27th edition, p. 349 Application

309. A patient is taking steroid drug for his frequent attacks of asthma. Steroids have anti-
inflammatory effects and also regulate gene expression. The effects of steroids are mediated
by DNA elements found in which transcription control regions of the gene?
A. Coding region C. Promoter proximal elements
B. Distal regulatory elements D. CAAT and TATA boxes
Harper's 27th edition, 354 - 356 Application

310. In beta thalassemia, the beta-globin gene is severely underexpressed resulting in diminished
synthesis of the beta-chain of hemoglobin. This is a consequence of which
posttranscriptional modification of primary RNA transcript?
A. Faulty exon-intron splicing
B. Addition of poly A tail in the 3' end
C. Incorporation of 7-methylguanosine cap in the 5' end
D. RNA editing in the coding region
Harper's 27th edition, p.362 Application

311. The genetic code is affected by insertion or deletion of nucleotides in DNA resulting in
frameshift mutation. Which characteristic of the genetic code is affected by this event?
A. Universal C. Nonoverlapping
B. Degenerate D. Unambiguous

Page 31 of 71
Page 32 of 71
Biochemistry

Harper's 27th edition, p.366, 370 - 371 Application

312. Chloramphenicol, a broad spectrum antibiotics, is known to inhibit peptidyltransferase


during translation. What step in protein synthesis is affected by chloramphenicol?
A. Formation of initiation complex.
B. Elongation of the polypeptide chain.
C. Termination of protein synthesis.
D. Posttranslational modification
Harper's 27th edition, p. 374 Recall

313. The elongation stage of protein synthesis is a multistep process which can be inhibited by
antibiotics. What will happen when the translocation step is inhibited by streptomycin?
A. Incoming aminoacyl-tRNA cannot bind to the A site
B. Small ribosomal subunit is unable to dissociated from the large ribosomal subunit.
C. No peptide bond synthesis can take place.
D. Peptidyl-tRNA cannot shift from the A site to the P site.
Harper's 27the edition, p. 375 Application

314. Some patients receiving methotrexate for treatment of cancer developed drug resistance.
What could be the reason for these?
A. Methotrexate induces amplification of genes for dihydrofolate reductase.
B. The chemotherapeutic agent inhibits the transport of mRNA from nucleus to cytoplasm.
C. The drug produces instability to the newly synthesized mRNA.
D. It promoted the use of alternative transcription start site.
Harper's 27th edition, p. 399 Application

315. Genomic technology oftentimes use probes. What can these molecules do?
A. They allow visualization of specific DNA fragments.
B. They are used to detect the presence of a specific fragment of DNA or RNA.
C. They amplify specific DNA sequences.
D. They are used to identify restriction sites.
Harper's 27th edition pp. 409, 420 Recall

316. Oxygen toxicity can be attributed to the destructive effects of free radicals generated. These
free radicals are removed by which of the following enzymes?
A. NADPH-cytochrome P450 reductase C. Superoxide dismutase
B. Peroxidase D. Catalase
Harper's 27th edition, pp. 97 - 98 Recall

317. A suicide victim suffered from carbon monoxide poisoning when he was trapped in the
garage with car engine running. Which component of the respiratory chain is inhibited by
the poisonous gas?
A. NADH dehydrogenase of Complex I
B. Succinate dehydrogenase of Complex II
C. Cytochrome c oxidoreductase of Complex III
D. Cytochrome oxidase of Complex IV
Harper's 27th edition, p. 107 Recall

318. People with bronchial asthma usually has hyperactive airway bronchoconstriction because
of inflammation. Which of the following eicosanoid is responsible for this?
A. leukotrienes C. thromboxanes
B. prostaglandins D. prostacyclins
Harper's 27th edition, p. 122 Application

319. A premature infant had respiratory distress syndrome after birth. Which of the following is a
major constituent of the lung surfactant?
A. distearoyl cephalin C. diarachidonyl plasmalogen
B. dioleyl lipositol D. dipalmitoyl lecithin
Harper's 27th edition, p. 125 Recall

320. Which of the following glycolipids is of considerable biologic interest because it serves as
the receptor in human intestine for cholera toxin?
A. sulfatide C. glucocerebroside
B. ganglioside (GM3) D. globoside

Page 32 of 71
Page 33 of 71
Biochemistry

Harper's 27th edition, p. 126 Recall

321. Which of the following steroids although NOT derived from cholesterol acquire antirachitic
property when irradiated with ultraviolet light?
A. cholic acid C. ergosterol
B. cholecalciferol D. corticosterone
Harper's 27th edition, p. 127 Recall

322. Which of the following glycolipids is of considerable biologic interest because it serves as
the receptor in human intestine for cholera toxin?
A. sulfatide C. glucocerebroside
B. ganglioside (G

323. How will carnitine deficiency affect the metabolism of glucose?


A. It will lead to hypoglycemia.
B. There is decreased synthesis of NADPH.
C. It will promote glycogenesis
D. There is increased synthesis of glucuronic acid
Harper's 27th ed., p 187 - Application

324. A patient suffering from hypoalbuminemia will affect the transport of which lipid in the
plasma?
A. cholesterol C. unesterified fatty acid
B. triacylglycerol D. phospholipid
Harper's 27th ed., 187 - Application

325. Lack of carnitine in the mitochondria will directly and immediately affect which aspect of
fatty acid oxidation?
A. activation of fatty acid
B. transport of fatty acid into the mitochondrial matrix
C. oxidation of fatty acyl CoA
D. formation of acetyl CoA
Harper's 27th ed., 187 - Application

326. Infants born without peroxisomes will suffer from neurologic symptoms because of the
accumulation of what type of fatty acid?
A. short chain saturated fatty acid
B. medium chain saturated fatty acid
C. long chain saturated and unsaturated fatty acid
D. very long chain unsaturated fatty acid
Harper's 27th ed, p 194 - Recall

327. The basic form of ketosis occurs in starvation which involves depletion of carbohydrates
coupled with increased:
A. mobilization of free fatty acid
B. fatty acid synthesis
C. triacylglycerol synthesis
D. cholesterol metabolism
Harper's 27th ed., 194 - Application

328. A patient taking aspirin because of chronic arthritis will have decreased synthesis of which
of the following?
A. leukotrienes C. prostaglandin
B. ketone bodies D. glycosphingolipid
Harper's 27th ed., 196 - Application

329. Deficiency of biotin will result in decreased formation of which of the following?
A. acyl carrier protein C. phosphatidic acid
B. malonyl CoA D. ceramide
Harper's 27th ed., 197 - Application

Page 33 of 71
Page 34 of 71
Biochemistry

330. Essential fatty acid deficiency may be exacerbated by increased intake of which of the
following foodstuffs?
A. margarine C. coconut oil
B. butter D. vegetable oil
Harper's 27th ed., 203 - 204 - Application

331. Which of the following enzymes that degrade phosphoglycerides is one of the major toxins
secreted by pathogenic bacteria?
A. Phospholipase A1 C. Phospholipase A2
B. Phospholipase C D. Phospholipase D
Harper's 27th ed., p. 212 - Recall

332. Enlarged liver and spleen, erosion of long bones, mental retardation in infants are seen in
which of the following lysosomal storage diseases?
A. Tay Sach's disease C. Gaucher's disease
B. Fabry's disease D. Farber's disease
Harper's 27th ed., 215 - Application

333. Patients with lipoprotein lipase deficiency will affect lipoprotein metabolism in what way?
A. Increased formation of lipoprotein remnants
B. Increased degradation of VLDL
C. Decreased synthesis of HDL
D. Decreased triglyceride hydrolysis in chylomicron
Harper's 27th ed., 219 - 220 - Application

334. Administration of simvastatin will decrease the activity of which of the following enzymes
involved in cholesterol synthesis?
A. HMG-CoA synthase C. Squalene oxidocyclase
B. HMG-CoA reductase D. Mevalonate kinase
Harper's 27th ed., p 231 - Recall

335. During protein analysis a new amino acid was found which has a pK3 value, 3 units lower
than that of cysteine. This so-called 21st amino acid is:
A. aluminocysteine C. manganocysteine
B. selenocysteine D. mangesimosysteine
Harper's 27th ed., p. 14 - Recall

336. A protein isolated from a cancerous tissue absorbs visible light in the region of 280 nm.
Which amino acid component of the protein is responsible for this?
A. glycine C. valine
B. arginine D. tryptophan
Harper's 27th ed., p. 18 - Recall

337. Hemoglobin has the ability to neutralize protons released by carbonic acid. Which of the
following amino acids is responsible for this?
A. lysine C. histidine
B. glutamine D. isoleucine
Harper's 27th ed., p. 45 - Application

338. Proteins must be purified prior to analysis. Size exclusion chromatography separates
proteins based on their Stokes radius which is a function of:
A. molecular mass and shape
B. solubility characteristics
C. chemical interactions with other groups
D. denaturation and renaturation potential
Harper's 27th ed., p. 21 - 22 -- Application

339. Scurvy which is due to vitamin C deficiency results in the formation of defective collagen
characterized by:
A. decreased number of hydroxyproline and hydroxylysine
B. increased in the number of covalent cross-links
C. decreased synthesis of disulfide bridges
D. increased interactions with copper and other minerals
Harper's 27th ed., p 39 - Application

Page 34 of 71
Page 35 of 71
Biochemistry

340. Tissue aminotransferases reversibly interconvert which of the following three amino acids
and their corresponding alpha-keto acids?
A. serine, threonine and tyrosine
B. lysine, arginine and histidine
C. glycine, alanine and isoleucine
D. valine, leucine and isoleucine
Harper's 27th ed., p. 243 - Application

341. Which of the following is NOT a part of the stages occuring in urea biosynthesis:
A. transamination
B. transaminidation
C. oxidative deamination of glutamate
D. ammonia transport
Harper's 27th ed., 247 - Application

342. Deficiency of which of the following coenzymes will result in decreased transamination of
amino acids?
A. thiamine pyrophosphate C. pyridoxal phosphate
B. biotin D. tetrahydrofolate
Harper's 27th ed., 248 - Application

343. Transdeamination refers to the concerted reactions catalyzed by which of the following pair
of enzymes?
A. glutamate aminotransferase and glutamate dehydrogenase
B. glutamate dehydrogenase and glutamine synthetase
C. L-amino acid oxidase and cyseine desulfurase
D. Serine dehydrase and glutamate transaminase
Harper's 27th ed., 248 - Application

344. Of the six participating amino acids in urea synthesis which of the following functions solely
as an enzyme activator?
A. ornithine C. citrulline
B. argininosuccinate D. N-acetylglutamate
Harper's 27th ed., p. 249 - Recall

345. Ammonia intoxication impairs neuronal functions of the brain because it:
A. impairs the synthesis of glutamine
B. inhibits the activation of asparagine synthetase
C. depletes the levels of alpha-ketoglutarate of the Kreb's cycle
D. produces respiratory alkalosis
Harper's 27th ed., p. 248 - Application

346. Which of the following chemical reactions typically initiates amino acid catabolism?
A. glutamate dehydrogenation C. transmethylation
B. transamination D. transulfuration
Harper's 27th ed., p. 254 - Recall

347. Alkaptonuria, an inborn error of tyrosine metabolism, is characterized by darkening of the


urine on exposure to air. This is due to oxidation of excreted:
A. homogentisic acid C. maleylacetoacetate
B. hydroxyphenylpyruvate D. fumarate
Harper's 27th ed., p. 259 - Application

348. Maple syrup urine disease is a metabolic disorder of branched-chain amino acid catabolism.
This is due to which of the following?
A. lack of leucine, isoleucine and valine aminotransferase complex
B. defective alpha-keto acid decarboxylase
C. mutation of dihydrofolate reductase
D. inability to attached the biotin coenzyme
Harper's 27the ed., p. 265 - Application

349. S-adenosylmethionine is important in the body because it is involved in which of the


following reactions?

Page 35 of 71
Page 36 of 71
Biochemistry

A. transulfuration C. methylation
B. acetylation D. conjugation reaction
Harper's 27th ed., p 274 - Recall

350. Which is a mechanism of oncogene activation? page 794 , MPL = 1


A. chromosome translocation C. promotion
B. initiation D. proliferation

351. TRUE of oncogene activation of chronic myelogenous leukemia: page 794, MPL= .25
A. It involves gene re-arrangement of chromosome 9 and 14
B. The mutation results to fusion of BCR-ABL gene
C. It is a recessive type of mutation
D.It results to overexpression of c-myc.

352. Stage of chemical carcinogenesis which is genotoxic and irreversible:


page 790, MPL= 1
A. initiation C. progression
B. promotion D. mutation

353. Philadelphia chromosome in chronic myelogenous leukemia is due to:


page 794, MPL= .5
A. gene rearrangement of chromosome 9 and 22
B. fusion of myc-ras
C. activation of c-myc
D. translocation of chromosome 8 and 14

354. Which of the following molecules function to transfer information from the nucleus to the
cytoplasm? (page 309, MPL=1)
A. DNA C. tRNA
B. mRNA D. proteins

355. Three types of RNA involved in comprising the structural and functional core for protein
synthesis, serving as a template for translation, and transporting amino acid, respectively,
are:
(Page 341, MPL=0.5)
A. mRNA, tRNA, rRNA C. rRNA, tRNA, mRNA
B. tRNA, mRNA, rRNA D. rRNA, mRNA ,tRNA

356. What is added to the 3'-end of many eukaryotic mRNAs after transcription?
(Page 356, MPL=0.75)
A. introns
B. a poly A tail A
C. a cap structure, consisting of a modified G nucleotide
D. the trinucleotide 5'-CCA
357. The regions of DNA in a eukaryotic gene that encode a polypeptide product are called:
(Page 352, MPL=1)
A. hnRNAs C. enhancers
B. exons D. leader sequences

358. A molecule containing sequences from two different species is known as a


(Page 419, MPL = 1)
A. clone
B. cosmid
C. plasmid
D. chimeric molecule

359. The sequence of a gene that is transcribed but excised before translation:
(Page 365, MPL = 1)
A. enhancer
B. exon
C. intron
D. repressor

Page 36 of 71
Page 37 of 71
Biochemistry

360. Which of the following statements is FALSE regarding a genomic library?


(Page 408, 409) MPL = 0.5
A. cDNA libraries are represented only by exons
B. genomic DNA libraries are represented only by introns
C. it is a collection of cloned fragments that represents the entire genome
D. libraries are prepared by performing partial digestion of total DNA with a restriction
enzyme

361. Which of the following statements is FALSE regarding the genetic code?
(Page 366, MPL = 0.5)
A. Some amino acids are encoded by several codons
B. In general, the third nucleotide in a codon is less important than the first two in coding
for a specific amino acid
C. It is universal with the exception of tRNA molecules in mitochondria
D. The reading of the genetic code may involve overlap of codons in some instances

362. The genetic code is said to have this property because multiple codons code for the same
amino acid. (Page 366, MPL = 1)
A. degenerate
B. unambiguous
C. universal
D. no punctuation

363. POMC gives rise to all of the following substances, except:


[p. 88] MPL- 1.0
A. ACTH
B. endorphins
C. α-MSH
D. renin

364. Which of the following structural motifs is important for the transcriptional activation
produced by some steroid-bound hormones? [pp. 393-396] MPL- 1.0
A. zinc fingers
B. gly-x-y
C. immunoglobulin folds
D. EF hands

365. Irreversible activator of adenylyl cyclase causing ribosylation of the α stimulatory - subunit of the
G-protein: [p. 567] MPL- 0.5
A. pertussis toxin
B. phosphodiesterase
C. cholera toxin
D. calcium

366. Catecholamines undergo oxidative deamination by the action of the following


enzyme: [25th ed., pp. 590-591] MPL – 0.5
A. tryptophan-5hydroxylase C. COMT
B. MAO D. tyrosine hydroxylase

367. Estrogenic hormone which is predominant during pregnancy.


[p. 450] MPL – 1.0
A. estradiol B. estriol C. estrone D. diethylstilbestrol

368. Leprechaunism is a rare condition characterized by elfin facies, low birth weight, decreased
muscle mass and subcutaneous fat due to which of the following defects?
[25th ed., pp. 622-623] MPL- 0.5
A. growth hormone deficiency
B. thyroid hormone receptor defect
C. growth hormone receptor defect
D. insulin receptor defect

369. Growth hormone is synthesized and stored in large amounts in the anterior pituitary gland.
Which of the following statements accurately describes GH?
[25th e4d., pp. 551-553] MPL- 0.5

Page 37 of 71
Page 38 of 71
Biochemistry

A. it is secreted continuously
B. it stimulates cartilage and bone growth via somatomedin C
C. it has a proinsulin-like effect in addition to its other actions
D. synthesis is stimulated by the action of somatostatin

370. In congenital adrenal hyperplasia involving 21-hydroxylase deficiency, which of the following
is true? [25th ed., p.587] MPL- 0.5
A. cortisol accumulates
B. 17-OH-progesterone is an effective treatment
C. most patients suffer "salt-wasting" due to lack of aldosterone
D. levels of ACTH in blood decreased

371. Which of the following is a direct effect of parathyroid hormone in regulating calcium
metabolism? [25th ed, p. 570] MPL – 0.5
A. increases bone resorption
B. increases intestinal absorption of calcium
C. decreases renal excretion of calcium
D. decreases bone mineralization

372. The bond that links together complementary base pairs along DNA strands :
A. hydrogen
B. glycosidic
C. phosphodiester
D. peptide
(Page 303, MPL = .5)

373. Poly A tail and 7 methyl guanosine cap are features of:
A. DNA
B. mRNA
C. tRNA
D. rRNA
(Page 309, MPL = .5 )

374. DNA synthesis occurs during this phase in the cell cycle :
A. G1 C. G0
B. G2 D. S phase
(Page 333, MPL = .5)

375. This drug blocks the reduction of dihydrofolate to tetrahydrofolate:


A. allopurinol C. methotrexate
B. colchicine D. sulfa drugs
(Page 296, MPL= .5)

376. A blot transfer procedure for RNA analysis. (Page 409, MPL = 1)
A. Southern blot C. Northern blot
B. Western blot D. Southwestern blot

377. A manual enzymatic method of DNA sequencing. (Page 410, MPL = 0.5)
A. Sanger’s method
B. Restriction fragment length polymorphism
C. Maxam-Gilbert method
D. Polymerase chain reaction

378. This is a method of amplifying a target sequence of DNA. (Page 410, MPL = 1
A. Flourescence in situ hybridization C. Sanger’s method
B. Polymerase chain reaction D. Microarray technology

379. A two year old boy, SHORT for his age, presented with leg edema and abdominal swelling.
He was weaned from breast milk at one year of age, because of the coming of another
baby. He was fed rice or lugaw everyday mixed with condensed milk. The edema is caused
by:
A. Lack of protein in his diet
B. Excessive intake of water
C. Kidney failure

Page 38 of 71
Page 39 of 71
Biochemistry

D. Presence of intestinal parasites

Recall
Reference: Harper's Illustrated Biochemistry 27 th ed., Murray, Granner & Rodwell 2006
The macgraw-Hill Companies, Inc., p. 241, par. 1, p. 487, par. 3

380. Catabolism of amino acids requires the participation of transaminase enzymes. What
Coenzyme form of vitamin B is an important participant in this reaction?
A. Thiamin pyrophosphate
B. Nicotinamide adenine dinucleotide
C. Tetrahydrofolic acid
D. Pyridoxal phosphate

Recall
Reference: Harper's Illustrated Biochemistry 27th ed., p. 247 par. 2 , p. 498 - 499

381. In marasmus, the affected person has very little fat reserves and there is wasting of
Muscles. The person is said to be in :
A. Positive nitrogen balance
B. Negative nitrogen balance
C. Nitrogen balance
D. Equilibrium

Recall
Reference: Harper's Illustrated Biochemistry 27th ed., p. 487 par. 3

382. Persons who consume large amounts of unleavened whole-wheat products may
Experience calcium deficiency because:
A. Wheat is poor in calcium
B. Wheat contains phytic acid which prevents calcium absorption
C. The wheat is deficient in vitamin D
D. Absorbable form of calcium is not generated

Application
Reference: Harper's Illustrated Biochemistry 27th ed., p. 485 par. 5

383. An elderly woman is believed to be suffering from vitamin B12 deficiency. Which of the
Following can be the basis for this statement?
A. She has been diagnosed to be deficient in cobalt
B. She is a vegan
C. She has methylmalonic aciduria
D. Any of the above

Application
Reference: Harper's Illustrated Biochemistry 27 th ed., p. 499-500

384. The patient was in coma. Since diabetes was ruled out, how does one explain the
condition?
A. Patient has ammonia intoxication because of the liver pathology
B. Coma is due to marked ketosis
C. Acid-base balance of the patient is disturbed
D. There is increase urea production

Application
Reference: Harper's Illustrated Biochemistry 27 th ed., p. 248 par. 4

385. The obese matron was instructed by her physician to alter her diet and eat only protein
foods to bring down her weight. The matron was afraid however that the absence of
carbohydrates in her meals will cause hypoglycemia and give her more problems. The
physician was certain she does not have to worry about this because:
A. She's going to cheat on her diet anyway

Page 39 of 71
Page 40 of 71
Biochemistry

B. The excess body fat will be converted to glucose


C. Amino acids in excess are converted to glucose
D. She has a lot of glycogen slores

Application
Reference: Harper's Illustrated Biochemistry 27 th ed., p. 172 par. 2, p. 175 summary, P.
269 summary

386. Deficiency of iron results in which of the following?


A. Failure to metabolize proteins
B. Heme-containing enzymes remain inactive
C. There is liver dysfunction
D. Individual goes into ATP production using fats

Recall
Reference: Harper's Illustrated Biochemistry 27th ed., p. 504 Table

387. An infant with genetic deficiency of glucose-6-phosphatase also known as von Gierke's
disease will exhibit this clinical sign:
A. Poor exercise tolerance
B. Muscle dystrophy
C. Hypoglycemia & lactic acidemia
D. Glycogen with few branch points

Recall
Reference: Harper's Illustrated Biochemistry 27th ed., p. 166 Table

388. In the absence of creatine kinase enzymes, an athlete would have problem in:
A. Oxidizing creatine
B. Releasing ATP from biological oxidation
C. Aerobic glucose oxidation
D. Generating ATP reserves for muscle contraction

Recall
Reference: Harper's Illustrated Biochemistry 27th ed., p. 583 par. 1-3

389. A young woman already in her fifth month of a carbohydrate and lipid-free diet was advised
by her physician to increase the amount of fats in her diet because she appears to be
suffering from vitamin A deficiency despite her regular vitamin intake. Which of the
following complaints presented by the woman supports the diagnosis?
A. She has lesions of the corner of the mouth
B. Her wounds take longer time to heal
C. She finds it hard to see in the dark when in a movie theater
D. She has this sensation of "burning feet".

Application
Reference: Harper's Illustrated Biochemistry, 27 th edition, p. 491, par. 4, line 3

390. Tests done on a two month-old baby showed deficiency in enzymes of the Uronic Acid
Pathway. What clinical manifestation may be observed?
A. Elevated urinary glucuronic acid
B. Yellow coloration of the skin
C. High direct-reacting bilirubin in plasma
D. Accumulation of heme

Application
Reference: Harper's Illustrated Biochemistry, 27 th edition, p. 289 par. 2, p. 290 par. 2

391. A 35 year old Muntinlupa inmate was confined to 6 weeks of isolation because of bad
behavior. Although he was given regular meals, he stayed in the underground cell for the
duration of the 6 weeks. If the prisoner became deficient in vitamin D because of this, what
signs should the attending physician look for? In the inmate:
A. There may be kidney stones formation
B. The inmate may complain of tiredness

Page 40 of 71
Page 41 of 71
Biochemistry

C. He can easily be fractured


D. Serum calcium will be extremely low

Application
Reference: Harper's Illustrated Biochemistry, 27 th edition, p. 492 - 493

392. Thiamine deficiency can result in Wernicke- Korsakoff's syndrome. The absence of the
vitamin prevents the conversion of pyruvate to acetyl coa on a high carbohydrate diet.
Which compound would be found to be high in concentration in the plasma?
A. Glucose
B. Lactate & pyruvate
C. Acetyl coa
D. Alcohol

Application
Reference: Harper's Illustrated Biochemistry, 27 th edition, p. 496- 497

393. Important in the synthesis of collagen, this vitamin serves as cofactor of the mixed function
oxygenase enzymes, proline & lysine hydroxylases:
A. Vitamin B1
B. Vitamin B2
C. Vitamin C
D. Folic acid

Recall
Reference: Harper's Illustrated Biochemistry 27th ed., p. 242 par. 3, line 9

394. Unlike other vitamins, niacin is a semi-essential vitamin because it can be synthesized from
this amino acid:
A. Proline
B. Phenylalanine
C. Tryptophan
D. Histidine

Recall
Reference: Harper's Illustrated Biochemistry 27th ed., p. 498, par. 1

395. Activity of the aldose reductase enzyme in the lens of the eye increases in the presence of
high concentration of glucose. This initiates the conversion of glucose to a compound that
has been linked to osmotic damage and cataract formation. What compound is this?
A. Mannitol
B. Galactitol
C. Ribitol
D. Sorbitol

Recall
Reference: Harper's Illustrated Biochemistry 27th ed., p. 185 last par.

396. A patient was tested for transketolase activity of the erythrocyte lysate. Results showed
markedly low activity of the enzyme, consistent with the physicians impression of
deficiency of this vitamin:
A. Vitamin B2
B. Vitamin B3
C. Vitamin B5
D. Vitamin B1

Application
Reference: Harper's Illustrated Biochemistry 27th ed., p. 497, par. 1

397. Inorganic iron is best absorbed by the body in the ferrous form. What is the rationale
behind the addition of vitamin C in many iron preparations?
A. Vitamin C can take the place of iron
B. Absorption of other minerals is prevented
C. Vitamin C chelates iron

Page 41 of 71
Page 42 of 71
Biochemistry

D. Vitamin C keeps the iron in reduced (ferrous) state

Application
Reference: Harper's Illustrated Biochemistry 27 th ed., p. 485 - 486

398. In a routine medical check up, an executive showed elevated plasma levels of lactic acid
dehydrogenase enzymes 1 & 2 and low levels of LDH 5. This prompted his physician to
refer him to a cardiologist because:
A. Low levels of LDH 5 indicated a clogged vessel
B. It is part of the medical check up
C. Increase levels of LDH 1 & 2 is indicative of myocardial infarction
D. Angioplasty was needed

Application
Reference: Harper's Illustrated Biochemistry 27 th ed., p. 57

399. A tourist from the Caribbean fell ill and was diagnosed to have malaria. He was given
primaquine upon admission to the hospital and was thought to be recuperating when he
showed signs of hemolytic anemia. Tests showed he had glucose 6- phosphatase deficiency.
What is the biochemical explanation for the hemolysis?
A. The malarial parasite had invaded the rbc
B. Inactivity of the Hexose Monophosphate Shunt prevented the NADPH - reduction of
membrane glutathione
C. The drug oxidized membrane proteins
D. Absence of G6PD initiated release of peroxides

Application
Reference: Harper's Illustrated Biochemistry 27 th ed., p. 177 par. 2, p. 183 par. 2

A. Metabolic screening of a newborn showed absence of galactose uridyltransferase


enzyme. What should be the dietary recommendation?
A. Galactose should not be included in the diet
B. Galactose - free and lactose -free milk should be given
C. Newborn should be breastfed
D. Liquid protein preparation instead of milk should be given

Recall
Reference: Harper's Illustrated Biochemistry 27 th ed., p. 186 par. 2

400. A born-again vegetarian was diagnosed to be suffering from vitamin B12 deficiency. Which
Of the following suggestions should be followed?
A. The vegetarian should include meat in the diet
B. He should be tested for pernicious anemia
C. The born-again can take in vitamin B12 preparation to meet the requirement
D. Intrinsic factor should be released from bonding

Recall
Reference: Harper's Illustrated Biochemistry 27 th ed., p. 499 - 500

401. A woman had just given birth by ceasarean section. For fast recuperation she should be
Advised to:
A. Eat nutritious and protein-rich foods
B. Rest in bed until the incision heals
C. Include more carbohydrates in the diet
D. Drink a lot of water

Recall
Reference: Harper's Illustrated Biochemistry 27 th ed., p. 487 - 488

Questions # 402-404

A 32 year old married woman consulted at the OPD because of increasing thirst and frequent
urination. She is 5 feet 2 inches tall and weighs 168 lbs.

Page 42 of 71
Page 43 of 71
Biochemistry

402. Which of the following statements best described the metabolic status in this patient?
a. Her energy expenditure is greater than food intake hence she is diuresing
b. She is obese and she has increased mortality
c. The (BMR) basal metabolic rate of this patient is higher compared to a man of the same
body weight
d. She is frequently urinating because she has increased protein reserves

Application
Reference: Harper's Illustrated Biochemistry 27 th ed., p. 486

403. The BMI of the patient is:


A. 25
B. 27
C. 29
D. 31

Application
Reference: Harper's Illustrated Biochemistry 27 th ed., p. 486 par. 2

404. You want to give this patient a 1,500 cal/day diet because you want her to lose weight. How
Do you formulate an appropriate plan for this patient?
A. Carbohydrate - 55% Fats = 35%
Protein = 10%

B. Carbohydrate - 30% Fats = 20%


Protein = 50%

C. Carbohydrate - 0% Fats = 40%


Protein = 60%

D. Carbohydrate - 40% Fats = 40%


Protein = 20%

Application
Reference: Harper's Illustrated Biochemistry 27 th ed., p. 132

405. The edema of a patient with kwashiorkor should respond best when prescribed with:
A. Diet rich in protein
B. Diuretics
C. Bed rest
D. Virgin coconut oil

Application
Reference: Harper's Illustrated Biochemistry 27 th ed., p. 487

406. A 32 year old physician suffered a 3rd degree burn. He stayed in the hospital for almost one
month. What can be said of his metabolic condition
A. The excretion of nitrogenous compounds is less than the dietary intake
B. Negative nitrogen balance
C. Positive nitrogen balance
D. There is net retention of the nitrogen in the body as protein

Application
Reference: Harper's Illustrated Biochemistry 27 th ed., p. 487

407. The human body, even at rest, uses up energy. This amount of energy expenditure is
termed as:
A. Respiratory Quotient
B. Basal Metabolic Rate
C. Respiratory Burst
D. Body Mass Index

Recall
Reference: Harper's Illustrated Biochemistry 27th ed., p. 486

Page 43 of 71
Page 44 of 71
Biochemistry

408. Uptake of glucose in these tissues is controlled by the hormone insulin:


A. Liver & kidneys
B. Adipose tissue & muscle
C. Muscle & intestines
D. Liver & rbc

Recall
Reference: Harper's Illustrated Biochemistry 27 th ed., p. 140, par. 3, line 1

409. Between meals, the blood glucose concentration of the body is maintained through
These metabolic pathways:
A. Glycolysis & gluconeogenesis
B. Glycogenesis & glycogenolysis
C. Glycolysis & glycogenolysis
D. Glycogenolysis & gluconeogenesis

Recall
Reference: Harper's Illustrated Biochemistry 27 th ed., p. 134, par. 4, line 8

410. During continuous anaerobic physical activity, e.g. When working out in a gym, this
metabolic pathway is used by the muscles
A. Gluconeogenesis
B. Glycolysis
C. Hexose Monophosphate Shunt
D. Glycogenesis

Recall
Ref. Harper's Illustrated Biochemistry 27th ed., p. 151, p. 158

411. When tested, a young woman, complaining of fatigability and poor exercise tolerance was
found to have only one sixth the amount of carnitine found in others who had no muscle
disease. This means that the above symptoms she is experiencing is due to:
A. Inability of the body to synthesize lipids
B. Inhibition of ATP production
C. Failure of fatty acids to enter - oxidation
D. Impairment of the mitochondria

Recall
Ref. Harper's Illustrated Biochemistry 27th ed., p. 187 par. 4

412. Newborn infants can generate more heat needed by the body because they contain
Thermogenin, an uncoupling protein. This protein exerts its effect by:
A. Increasing the amount of substrate oxidized
B. Preventing heat loss
C. Dissociating respiratory chain oxidation from ATP synthesis (phosphorylation)
D. Activating respiratory chain enzymes

Application
Reference: Harper's Illustrated Biochemistry 27 th ed., p. 107, par. 4, p. 228 - 229

413. Erythrocytes are hemolyzed when there is impairment of the Pentose Phosphate Pathway
and this is manifested in a person as hemolytic anemia. This defect is due to:
A. Lack of NADPH from the Pentose Phosphate Pathway
B. Non-synthesis of ATP
C. Failure to release ribose
D. Defect in the membrane transport

Application
Reference: Harper's Illustrated Biochemistry 27 th ed., p. 183- 184, par.1

414. A 30 year old patient was diagnosed to be suffering from a mitochondrial disease that
specifically involves defect in mitochondrial energy transduction.

Page 44 of 71
Page 45 of 71
Biochemistry

Which of the following signs/symptoms will be evident?


A. High body mass index (BMI), hyperactivity, increased appetite
B. Hyperventilation, clammy sweat
C. Muscle weakness, excessive perspiration, and elevated BMR (Basal metabolic
Rate)
D. Lack of appetite, hypoventilation

Application
Reference: Harper's Illustrated Biochemistry 27 th ed., p. 100, par. 2

415. A newborn infant tested positive in the phenylketonuria screening. The attending
pediatrician ordered a change in the infant milk to one which contains negligible amount of:
A. Lactose
B. Proteins
C. Fats
D. Glucose

Application
Reference: Harper's Illustrated Biochemistry 27 th ed., p. 259, par. 3

416. A drug used as an adjunct in the treatment of obesity inhibits pancreatic lipase. Following its
intake, which of the following is expected to be excreted in great amount in the feces?
A. Cholesterol
B. Triacylglycerols
C. Free fatty acids
D. Bile salts

Application
Reference: Harper's Illustrated Biochemistry 27 th ed., p. 483, par. 4, p. 484 diagram

417. In the mucopolysaccharidoses, glycosaminoglycans are not degraded due to the absence of
lysosomal hydrolases. Individuals suffering from this can be identified because of:
A. Excessive accumulation and urinary excretion of the proteoglycans
B. Presence of enzyme fragments in the urine
C. Release of the core proteins in urine
D. Increase turnover of affected cells

Recall
Reference: Harper's Illustrated Biochemistry 27 th ed., p. 555 par. 5

418. In the rare disease, analbuminemia, there is complete lack of serum albumin. The affected
Individual presents with this:
A. Extreme edema
B. Edema & presence of free fatty acids
C. Accumulation of triglycerides
D. Elevated plasma cholesterol

Recall
Reference: Harper's Illustrated Biochemistry 27 th ed., p. 592 par. 1, p. 187 par. 3 P. 711
Clinical Correlation , Textbook of Biochemistry, 5 th ed., Thomas Devlin, Editors, 2002 Wiley-
Liss

419. Metabolic screening was requested for a week-old newborn because the grandmother had
noticed the dark stains on the wet diaper whenever she attended the infant. Tests
identified the condition as alkaptonuria. What urinary metabolic should the physician
request in the laboratory tests?
A. Tryptophan
B. Tyrosine
C. Homogentisic acid
D. Melanin

Recall
Reference: Harper's Illustrated Biochemistry 27 th ed., p.259 par. 2
Textbook of Biochemistry, 5th ed., Thomas Devlin, p. 801, Clinical Correlation

Page 45 of 71
Page 46 of 71
Biochemistry

420. Skeletal muscles do not have the enzyme glucose-6-phosphatase. This means:
A. They cannot degrade glucose
B. Glucose cannot be synthesized
C. ATP cannot be generated
D. Glycogen is stored in big amount

Recall
Reference: Harper's Illustrated Biochemistry 27 th ed., p. 161, p. 167

421. L-xylulose reductase catalyzes the reduction of xylulose to xylitol. In affected individuals,
Absence of this enzyme results in:
A. Formation of cataracts
B. Failure to synthesize nucleic acids
C. Excretion of large amounts of urinary pentose
D. Presence of skeletal deformities

Recall
Reference: Harper's Illustrated Biochemistry 27 th ed., p. 184 par. 2

422. When there is restricted intake of carbohydrates as in starvation and uncontrolled diabetes
mellitus, ketosis results because of:
A. Change in ph of the body
B. Failure to oxidize glucose
C. Mobilization of fatty acids
D. Inactivity of buffer systems

Application
Reference: Harper's Illustrated Biochemistry, 27th ed., p. 194 last par. , p.195 , 2nd par.
(summary)

423. Measurement of hba1c or glycosylated hemoglobin provides valuable information for the
management of diabetes mellitus because it:
A. Is elevated in diabetes mellitus
B. Reflects the mean glucose concentration over the preceding 6 - 8 weeks
C. Is formed by reaction with hba
D. Is released when rbc dies

Application
Reference: Harper's Illustrated Biochemistry 27 th ed., p. 48, par.1

424. Inherited enzyme deficiencies of the Krebs-Henseleit cycle always present with this
observation:
A. Increase urea production
B. Formation of organic metabolites
C. Ammonia intoxication
D. Depletion of citrulline

Application
Referenece: Harper's Illustrated Biochemistry 27th ed., p. 251, par. 4

425. The urine-soaked diaper of a week old baby was noted by the mother to smell like burnt
sugar. Her concern turned into worry when the pediatrician told her the baby may be
suffering from maple syrup urine disease. To confirm this, activity of the keto acid
decarboxylase complex has to be measured. Which amino acids will have increased
concentration in plasma and urine?
A. Serine, threonine & hydroxyproline
B. Cysteine, cystine, methionine
C. Valine, leucine & isoleucine
D. Glycine, alanine & asparagine

Application

Page 46 of 71
Page 47 of 71
Biochemistry

Reference: Harper's Illustrated Biochemistry 27th ed., p. 265, par. 2

426. Calcium deficiency manifestations may also be seen in people who consume:
A. Large amounts of unleavened whole wheat products
B. High concentrations of fatty acids
C. High amounts of oxalates
D. All of the above

Application
Referenece: Harper's Illustrated Biochemistry 27th ed., p. 265, par. 2

427. Milk intolerance, common among Asian populations and characterized by an osmotic type
of diarrhrea following the intake of milk is attributed to deficiency of:
A. Pancreatic amylase
B. Lactase
C. Sucrase
D. Maltase

Recall
Ref. Harper's Illustrated Biochemistry 27th ed., p. 483, par. 1, line 3

428. Rate-limiting reaction in cholesterol synthesis which is inhibited by the "statins" group of
Drugs is catalyzed by this enzyme:
A. Cholesterol esterase
B. Squalene cyclase
C. Mevalonate kinase
D. Beta hydroxyl-beta methyl glutaryl coa reductase

Recall
Ref. Harper's Illustrated Biochemistry 27th ed., p. 231, par. 2

429. All the carbon atoms of cholesterol are synthesized from this common metabolite:
A. Alanine
B. Acetyl coa
C. Glucose
D. Palmitate

Recall
Ref. Harper's Illustrated Biochemistry 27th ed., p. 230, par. 3

430. Carbohydrates, lipids & proteins can be oxidized to acetyl coa or specific metabolites
intermediates using a final common pathway. The pathway functions in gluconeogenesis,
lipogenesis and interconversion of amino acids. This pathway is known as:
A. Glycolysis
B. Hexose Monophosphate Shunt
C. Citric Acid Cycle
D. Glucuronic Acid Pathway

Recall
Ref. Harper's Illustrated Biochemistry 27th ed., p. 145, par. 1

431. In the capillaries, hemoglobin releases oxygen and accepts carbon dioxide. The process
generates protons e which can then lower the blood ph. The buffering action of hemoglobin
will prevent this from taking place. What amino acid residues in hemoglobin are most
important in this shift in ph buffering effect at normal blood ph?
A. Arginine
B. Lysine
C. Histidine
D. Glutamate

Recall
Ref. Harper's Illustrated Biochemistry 27th ed., p. 43

Page 47 of 71
Page 48 of 71
Biochemistry

432. The highest number of energy released as ATP is obtained by complete oxidation of this
compound to carbon dioxide and water
A. An amino acid
B. A triglyceride
C. Glucose
D. A nucleic acid

Application
Ref. Harper's Illustrated Biochemistry 27th ed., p. 142, table 16-1, p.189, par.1

433. Laboratory tests done on a 3 year old boy showed excessive nucleic acid bases in the urine
and negligible activity of hgprtase. Which statement explains the results obtained?
A. Purine bases are very insoluble
B. Low enzyme activity is due to feedback inhibition
C. Salvage pathway of synthesizing nucleotides is inactive
D. Young cells utilize the in vivo method for synthesis

Application
Ref. Harper's Illustrated Biochemistry 27th ed., p. 308, par. 1

434. In myocardial infarction where a major coronary artery is occluded, ischemia results and a
lowered oxygen supply diminishes the amount of ATP available in the cell. This effect on
cellular energy is due to:
A. Release of enzymes
B. Lack of oxygen which slows down the mitochondrial electron transport chain
C. Inhibition of substrate level phosphorylation
D. Kind of occlusion

Application
Ref. Textbook of Biochemistry, 5th ed., Thomas Devlin, p. 591,
Clinical Correlation

435. The patient was diagnosed to be suffering from megaloblastic anemia. Results of urinary
tests showed 10 mg of formiminoglutamic acid (FIGLU), making the physician conclude that
the patient has folic acid deficiency. Which statement explains the diagnosis given?
A. Megaloblastic anemia is a unique feature of folic acid deficiency
B. Transfer of formimino group of histidine to tetrahydrofolate does not take place and
figlu is excreted
C. Figlu is a metabolite of folate
D. One carbon compounds are carried by folic acid

Application
Ref. Harper's Illustrated Biochemistry 27th ed., p. 254 par. 6

436. This is a correct statement regarding myoglobin:


A. A monomeric protein
B. Exhibits cooperativity
C. A tetrameric protein
D. Transports oxygen
MPL= 0.5(page:40)

437. This is a correct statement regarding hemoglobin:


A. Stores oxygen
B. Exhibits cooperativity
C. A stores CO2
D. A non-heme protein
MPL = 0.5 page: 40

438. The globin chains in hemoglobin and myoglobin functions as/to:


A. oxygen transporter
B. Provides hindered environment for the heme iron
C. inhibits allosteric modulators
D. suit the proteins’ physiologic roles

Page 48 of 71
Page 49 of 71
Biochemistry

MPL= 0.5 page: 41

439. Allosteric properties of hemoglobin is a result if its:


A. Primary structure
B. Quarternary structure
C. Tertiary structure
D. Heme moiety
MPL = 0.5 page 42

440. The color of heme is due to its:


A. Conjugated ring structure
B. Substituent side chains
C. Cyclic Structure
D. Iron group
MPL = 1.0 p. 271

441. The key regulatory enzyme in heme biosynthesis is:


A. ALA dehydratase
B. ALA synthase
C. Heme oxygenase
D. UDP-glucuronyltransferase
MPL =0.5 p. 272

442. Sugar with glycemic index less than 1:


A. Glucose
B. Fructose
C. Galactose
D. Lactose
MPL = 1.0 p. 474

443. Foods with low glycemic index:


A. cause less fluctuation in insulin secretion
B. Not ideal in treating diabetes mellitus
C. examples are starch
D. Glucose have glycemic index more than 1
MPL = 0.5 p. 474

444. Enzyme that catalyzes hydrolysis of starch:


A. Lipase
B. Amylase
C. protease
D. HCL
MPL = 1.0 p. 474

445. Clinical manifestation of lactose intolerance is due to:


A. Lactose presence in the large intestines
B. Bacterial fermentation of glucose
C. Bacterial fermentation of galactose
D. Lactase oversecretion of the small intestines
MPL = 0.25 p. 475

446. A sugar that is absorbed in the small intestine by active transport:


A. Maltose
B. Glucose
C. Fructose
D. Lactose
MPL = 1.0

447. This is the process of forming glucose from noncarbohydrate precursors like lactate, amino
acids and glycerol:
A. glycolysis
B. gluconeogenesis
C. glycogenesis
D. cori cycle

Page 49 of 71
Page 50 of 71
Biochemistry

MPL= 0.5 p. 133

448. Muscles glycogen cannot contribute directly to plasma glucose because it lacks this enzyme:
A. Glucose-6-phosphate dehydrogenase
B. Phosphoglucomutase
C. Glucose-6-phosphatase
D. Glucokinase
MPL=0.75 p. 142

449. Nearly all products of digestion of carbohydrates, fats and proteins are metabolized to what
common metabolite?
A. Acetyl CoA
B. pyruvate
C. Glucose-6-phosphate
D. lactate
MPL=0.5144

450. It is referred to as the final common pathway for the oxidation of carbohydrates, lipids and
proteins:
A. Embden-Meyerhof pathway
B. Hexose monophosphate shunt
C. Glucuronic acid pathway
D. Citric acid cycle
MPL=1.0p. 145

451. These two amino acids yield only Acetyl CoA on oxidation, and hence cannot be used for
gluconeogenesis:
A. Glycine and Lysine
B. Lysine and leucine
C. Valine and Leucine
D. Proline and Glycine
MPL=0.5p. 139

452. Interconversion of Glycine and Serine requires what coenzyme?


A. Niacin
B. Riboflavin
C. Pyridoxine
D. Folic acid
MPL=1.0p. 257

453. A major role of this vitamin is in the control of cell differentiation and turnover:
A. Vit A
B. Vit C
C. Vit E
D. Vit D
MPL=0.75 p. 491

454. This is the earliest sign of Vit A deficiency:


A. Inability to adapt to dim light
B. loss of sensitivity to green light
C. keratomalacia
D. keratinization of the lining of epithelial cells
MPL=0.5p. 491

455. What is the most important function of lipid in the adipose tissue and around internal
organs?
A. as electrical insulator
B. as thermal insulator
C. as source of energy
D. for lipid transport
MPL:75% p. 121

456. Which among the following is a complex lipid composed of fatty acid and shingosine and
carbohydrate?

Page 50 of 71
Page 51 of 71
Biochemistry

A. fats
B. oil
C. phospholipids
D. glycolipids
MPL:50% p. 121

457. Which eicosanoids promotes bronchoconstriction and play a part in asthma?


A. prostaglandins
B. leukotienes
C. Thromboxanes
D. Prostacyclin
MPL:50% p. 122

458. What is the process involved in the manufacture of margarine?


A. Hydrogenation
B. Hydroxylation
C. Dehydrogenation
D. Oxidation
MPL:50% p. 123

459. Which is considered as transcriptionally active chromatin


A. euchromatin
B. heterochromatin
C. constitutive chromatin
D. telomerase
MPL:75% p. 324

460. This is the noncoding intervening sequence


A. exon
B. intron
C. kinetochore
D. telomerase
MPL:75% p. 324

461. This is the enzyme responsible DNA unwinding during replication


A. DNA ligase
B. Helicase
C. DNA polymerase
D. Topoisomerase
MPL:75% p. 336

462. Eukaryotic RNA polymerase II synthesizes:


A. rRNA
B. mRNA
C. tRNA
D. snRNA
MPL:25% p. 351

463. What is the key tool in recombinant DNA research?


A. taq polymerase
B. restriction enzymes
C. Deoxyribonuclease
D. Primase
MPL:50% p. 403

464. The total DNA of a cell or tissue.


A. Genomic DNA library
B. cDNA library
C. introns
D. exons
MPL:50% p. 408

465. Immunoglobulin are mostly synthesized in the:


A. T-helper cell

Page 51 of 71
Page 52 of 71
Biochemistry

B. Natural killer cells


C. Plasma cells
D. macrophages
MPL:1.0 p. 599

466. Not true of an immunoglobulin:


A. it is made up of 2 L chains and 2 H chains
B. it has a Y or T shape
C. it has 4 variable regions
D. it has 2 constant regions in its entire structure
MPL:0.25 pp. 599-600

467. True of the variable region of an immunoglobulins:


A. it is located at the middle most part of the light and heavy chains
B. it is where the CDRs (complementary determining regions) are located
C. may form two or three antibodies which are identical to each other.
D. there are no hypervariable regions.
MPL:0.25 p. 601

468. The part of the antibody that binds to the antigen:


A. Fa,b
B. The antigen determinant site
C. The N-terminal of both the light and the heavy chain
D. The hypervariable regions of both the L and H chains
MPL:0.5 p. 601

469. The part of the antigen that binds to the antibody:


A. Fa,b
B. The antigen determinant site
C. The N-terminal of both the light and the heavy chain
D. The hypervariable regions of both the L and H chains
MPL:0.5 p. 601

470. A transcription factor involved in the stimulation of acute phase proteins


synthesis, is nuclear factor __________,
A. Gamma-B
B. Kappa-B
C. Alpha-B
D. Omega-B
MPL:0.25 p. 591

471. The C-reactive protein is so called as such, because it binds with the _________,
of Pnuemococci, haptoglobin, α, acid glycoproteises:
A. C-monomers
B. C-dimers
C. C-polypeptides
D. C-polysaccharides
MPL:0.25 p. 590

472. The proteins that stores iron in the cells, is:


A. hemosiderin
B. hemochromatine
C. ferritin
D. . transferring
MPL:0.25 p. 594

473. The most potent activator of platelet aggregation in thrombus formation is:
A. Collagen
B. TXA
C. ADP
D. fibrinogen
MPL:0.25 p.; 609

474. A type of thrombus that is formed in arteries is:

Page 52 of 71
Page 53 of 71
Biochemistry

A. disseminated fibrin deposits


B. white thrombus
C. red thrombus
D. yellow thrombus
MPL:1.0 p. 606

475. The chief constituent of fibrous part of plants, is:


A. Starch
B. Cellulose
C. Chitin
D. Peptidoglycan
MPL:1.0 p. 116

Which of the following carbohydrates do not have a structural functions?


A. Cellulose
B. Mucin
C. Chitin
D. Peptidoglycan
MPL:0.25 p. 119

476. The de novo pathway of purine biosynthesis utilized______ATPs:


A. 7
B. 8
C. 6
D. 5
MPL:0.5 p. 305

477. The ultimate hydrogen donor in deoxyribonucleotides formation is:


A. thioredoxin
B. NADPH
C. dihydrofolate
D. NADH
MPL:0.5 p. 306

478. Which of the following coenzymes is involved in electron transfer?


A. NAD
B. biotin
C. pyridoxal phosphate
D. tetrahydrofolic acid
MPL:0.5 p. 49

479. Which of the following statements about noncompetitive inhibitor is true?


A. it is a structural analog of the normal substrate
B. it binds to the active site of enzyme
C. Km is not change but Vmax is decreased
D. It can be reversed by adding more substrate
MPL:0.25 p. 68

480. Enzymes that are activated through covalent modification usually involved 75
phosphorylation of this amino acid residue in the active site.
A. phenylalanine
B. tryptophan
C. lysine
D. serine
MPL:0.5 p. 75

481. Carbon monoxide from vehicle exhaust inhibit mitochondrial ATP production by,
A. decreasing overall potential of the electron transport chain
B. uncoupling oxidative phosphorylation
C. disturbing proton gradient across intermembranous space and matrix
D. binding to cytochrome oxidase

482. Oxidation is the,


A. loss of electrons

Page 53 of 71
Page 54 of 71
Biochemistry

B. gain of electron
C. gain of protons
D. digestion of energy foods

483. The final electron acceptor is,


A. carbon dioxide
B. oxygen
C. water
D. hydrogen

484. Which of the following is responsible for the polarity of a strand of nucleic acid?
A. acid
B. base
C. sugar
D. phosphate

485. Nucleic acids are polymeric macromolecules composed of units of,


A. nitrogenous bases
B. nucleotides
C. negatively charge phosphates
D. nucleosomes

486. What is the bond that links nucleotides together to form a strand of nucleic acid?
A. glycosidic bond
B. phosphodiester bond
C. phosphoester bond
D. hydrogen bond

487. Which vitamin in the diet is required for de novo pathway of adenosine monophosphate
synthesis and whose deficiency may lead to purine deficiency, a rare disease in humans?
A. folic acid
B. nicotinic acid
C. pantothenic acid
D. vitamin C

488. Which of the following organs/tissues is capable of de novo purine biosynthesis?


A. brain
B. erythrocytes
C. polymorphonuclear leukocytes
D. liver

489. The lipooxygenase pathway synthesizes:


A. Prostaglandins
B. Leukotrienes
C. Thromboxanes
D. Lipid peroxides

490. Ketone bodies are oxidized for ATP generation via:


A. Beta-oxidation pathway
B. Cyclooxygenase pathway
C. Lipoxygenase pathway
D. Kreb’s citric acid cycle

491. Glycerol phosphate, the backbone of TAG, is synthesized in adipose tissue by:
A. phosphorylation of glycerol by glycerol kinase
B. phosphorylation of diacylglycerol
C. reduction of dihydroxyacetone phosphate
D. glycerol dehydrogenase

492. Which of the following is not true of the beta-oxidation of palmitic acid?
A. occurs in the matrix of mitochondria
B. two oxidation steps generate 5 mols of ATP per cycle
C. cleavage reaction occurs between the beta and gamma carbons
D. the cell conserves 129 moles ATP per mole of palmitic acid

Page 54 of 71
Page 55 of 71
Biochemistry

493. Glycolysis, glycogenesis and lipogenesis are stimulated by this hormone:


A. glucagons
B. cortisol
C. insulin
D. epinephrine

494. This hormone is synthesized and released immediately in its final form:
A. aldosterone
B. insulin
C. T3
D. DHT
MPL:1.0

495. Hormone derived from cholesterol


A. TSH
B. 1,25(OH)2-D3
C. Tyroxine
D. GH
MPL:1.0 p. 447

496. Which sugar derivatives polymerize to form structural polysaccharide?:


A. Sugar phosphate and sugar acid
B. Sugaramino acid and sugar sulfate
C. Sugar acid and sugar sulfate
D. Sugar amino and sugar acid

497. Digestability of sugar depends on the presence of this glycosidic linkage:


A. Alpha
B. Beta
C. Gamma
D. Theta

498. Which of the following disaccharides is best suited to provide the structural sugars?:
A. Sucrose
B. Lactose
C. Mannose
D. Cellubiose

499. Which of the following is a saturated fatty acid?:


A. Oleic
B. Linoleic
C. Arachidonic
D. Palmitic

500. Which of the following is a phospholipid?:


A. Ceramide
B. Cholesterol
C. Cephalin
D. Corticosteroid

Which of the following is a fatty acid?:


A. Succinic acid
B. Prostanoic acid
C. Glutamic acid
D. Butyric acid

501. Which of the following backbone will make the lipid saponifiable ?:
A. Isoprene nucleus
B. Steroid nucleus
C. Sphingosine
D. Prostanoic acid

Page 55 of 71
Page 56 of 71
Biochemistry

502. The presence of which fatty acid will make the membrane more fluid:
A. Myristic
B. Palmitic
C. Stearic
D. Lignoceric

503. Which of the following lipids will stabilize membrane?:


A. Cardiolipin
B. Cortisol
C. Cholesterol
D. Ceramide

504. Which of these is an essential amino acid?:


A. Serine
B. Tyrosine
C. Isoleucine
D. Glutamine

505. The simplest amino acid is:


A. Cysteine
B. Glycine
C. Serine
D. Valine

506. An example of an aromatic amino acid is:


A. Serine
B. Aspartic
C. Glutamic
D. Tryptophan

507. A protein exposed to denaturing agent becomes:


A. Antigenic
B. Inert
C. Undigestible
D. Reactive

508. The linkage between the nucleotides:


A. Glycosidic
B. Peptide
C. Methene
D. Phosphodiester

509. An important bond in base pairing:


A. Covalent
B. Glycosidic
C. Hydrogen
D. Phosphodiester

510. DNA strands that show high melting temperature contains:


A. High carbohydrate residues
B. High histone content
C. High G-C pairs
D. High supercoiling

511. Which purine analogue is useful in cancer chemotherapy?:


A. Propyl thiuracil
B. 1,3,7 trimethylxanthine
C. 6 mercaptopurine
D. 4 hydroxypyrazolopyramidine

512. Which vitamin is a purine derivative?:


A. Cyanocobalamine
B. Inositol
C. Carotene

Page 56 of 71
Page 57 of 71
Biochemistry

D. Niacin

513. Enzyme that hydrolyzes glycogen:


A. Hexokinase
B. Phosphorylase
C. Aconitase
D. Transketolase

514. A metabolic intermediate leading to glycogenesis, pentose phosphate pathway and


glycolysis:
A. Acetyl CoA
B. Citric acid
C. Glucose 6 phosphate
D. Pyruvate

515. This metabolite when transaminated leads to sugar amine pathway:


A. Fructose 6 phosphate
B. Glyceraldehyde 3 phosphate
C. Glucose 1 phosphate
D. Ribose 5 phosphate

516. This metabolite leads to the production of NAD in Embden Meyerhoff pathway:
A. Acetyl CoA
B. Glyceraldehyde 3 phosphate
C. Lactic acid
D. Pyruvate

517. This pathway sustains the energy requirement of mature RBC.


A. Kreb’s cycle
B. Pentose phosphate
C. Glycolysis
D. Anaplerotic

518. This pathway provides NADPH for lipogenesis


A. Kreb’s cycle
B. Phosphogluconic
C. Uronic
D. Cori

519. In the intestinal mucosal cells, triglyceride is:


A. Hydrolyzed
B. Esterified
C. Synthesized
D. Absorbed

520. Lipolysis will be formed with increased levels of this hormone:


A. Insulin
B. Epinephrine
C. Cortisol
D. Esatrogen

521. In beta oxidation, for every break of an ester bond (saturated) this much amount of ATP is
generated:
A. 23
B. 3
C. 5
D. 12

522. In fasting and starvation the level of these metabolites are increased:
A. Oxysteroid
B. Ketone bodies
C. Thyroxine
D. Insulin

Page 57 of 71
Page 58 of 71
Biochemistry

523. Arachidonic acid deficiency usually do not occur because of this enzyme:
A. Polymerase
B. Kinase
C. Desaturase
D. Ligase

524. Amino acids are transported intracellularly using this cycle:


A. Kreb’s cycle
B. Kreb’s – hanseleit cycle
C. Cori cycle
D. Gamma glutamyl cycle

525. The major product of nitrogen catabolism in humans is:


A. Uric acid
B. Creatinine
C. Urea
D. Ammonia

526. What urinary metabolite is useful in the assessment catecholamine induced hypertension?:
A. Hydroxyindole acetic acid
B. Phenylpyruvic acid
C. Vanillyl mandylic acid
D. Paraamino hippuric acid

527. The starting materials for porphyrin biosynthesis are:


A. Acetyl CoA and serine
B. Malonyl CoA and glutamine
C. Succinyl CoA and glycine
D. Amino acyl and methionine

528. Which of these is the key regulatory enzyme in heme biosynthesis?:


A. Ferrochelatase
B. ALA synthase
C. Pyrollase
D. Uroporphyrin decarboxylase

529. What is this organic acid that is used to conjugate bilirubin?:


A. Propionic acid
B. Glucuronic
C. Acetic acid
D. Butyric acid

530. Which of these enzyme deficiency produces abdominal pain with out skin manifestation?:
A. Coproporphyrinogen oxidase
B. Uroporphyrin synthase
C. Ferrochelatase
D. Pyrollase

531. Activity exhibited by enzymes when fully saturated:


A. Km
B. Vmax
C. ½ Vmax
D. Vo

532. CPK elevation is suggestive of:


A. Hepatitis
B. Cardiomyopathy
C. Pancreatitis
D. Bone tumor

533. Pyruvate kinase deficiency produces:


A. Hypoglycemia

Page 58 of 71
Page 59 of 71
Biochemistry

B. Hemolytic anemia
C. Hypercholesterolemia
D. Mal absorption

534. Which of these is a pyrimidine excretory metabolite?:


A. Ammonia
B. Urea
C. Uric acid
D. Allantoin

535. Which of these enzymes is important in DNA sequencing?

A. Ligase
B. Polymerase
C. Exonuclease
D. Kinase

536. A very anxious student hyperventilates an hour before the biochemistry board exam. What
is the effect of hyperventilation on his Pco 2 and blood pH?
A. Pco2 decreases and pH increases.
B. Pco2 decreases and pH decreases.
C. Pco2 increases and pH decreases.
D. Pco2 increases and pH increases.

Ans. A
Ref: Harper’s Biochemistry 24th ed. Pp. 18-19
MPL: 0.25

537. A protein rich in which of the following amino acids will provide the greatest buffering
capacity at physiologic pH?
A. Valine
B. Aspartic acid
C. Lysine
D. Histidine

Ans. D
Ref: Harper’s Biochemistry 24th ed. p.26
MPL: 0.75

538. Which of the following laboratory results would best indicate metabolic alkalosis?
A. Increased Pco2, increased pH, increased HCO3.
B. Normal Pco2, increased pH, decreased HCO3.
C. Increased Pco2, increased pH, normal HCO3.
D. Decreased Pco2, increased pH, normal HCO3.

Ans. A
Ref: Harper’s Biochemistry 24th ed. p.304
MPL: 0.75

539. What is the property of water that will contribute the most to its ability to dissolve
compounds?
A. Very few interactive forces in its structure.
B. Hydrogen bond formation between water and long-chain fatty acids.
C. Covalent bond formation between salt and water.
D. Hydrogen bond formation between water and biochemical molecules.

Ans. D
Ref: Harper’s Biochemistry 24th ed. p.15-17
MPL: 0.75

540. A 3-year old child is diagnosed to have Hurler’s syndrome. The diagnosis of Hurler’s
syndrome is MOST efficiently made by analyzing the patient’s DNA for:
A. A region of DNA that does not encode RNA.
B. Alternative forms of the L-Iduronidase gene.

Page 59 of 71
Page 60 of 71
Biochemistry

C. The entire set of genes in one leukocyte.


D. A nucleotide substitution in the L-Iduronidase gene.

Ans. B
Ref: Harper’s Biochemistry 24th ed. p. 678
MPL: 0.25

541. Which of the following statements regarding a double-helical molecule of DNA is TRUE?
A. All hydroxyl groups of pentoses are involved in linkages.
B. Bases are perpendicular to the axis.
C. Each strand is identical.
D. Each strand replicates itself.

Ans. B
Ref: Harper’s Biochemistry 24th ed. p.386
MPL: 0.75
542. A newborn baby has a sibling with sickle cell anemia and is at risk for this disease. The
appropriate diagnostic test for sickle cell anemia in this baby would include which of the
following?
A. DNA amplification
B. Hemoglobin antibodies
C. Red cell counting
D. DNA fingerprinting

Ans. A
Ref: Harper’s Biochemistry 24th ed. p. 468
MPL: 0.25

543. Which of the following would BEST describe polymorphism?


A. One phenotype, multiple genotypes
B. Non-random allele association
C. One locus, multiple abnormal alleles
D. One locus, multiple normal alleles

Ans. D
Ref: Harper’s Biochemistry 24th ed. p. 476
MPL: 0.25

544. What is that process that occurs at the 5-position of Cytidine and is often correlated with
gene inactivation?
A. Gene conversion
B. Sister chromatid exchange
C. Gene rearrangement
D. DNA methylation

Ans. D
Ref: Harper’s Biochemistry 24th ed. p.466
MPL: 0.75

545. Restriction Fragment Length Polymorphism (RFLP) analysis can only be used to follow the
inheritance of a genetic disease if:
A. The disease-causing mutation is at or closely linked to an altered restriction site.
B. Proteins of mutated and normal genes migrate differently upon gel electrophoresis.
C. Mutations are outside restriction sites so that cleaving still occurs.
D. mRNA probes are used in combination with antibodies.

Ans. A
Ref: Harper’s Biochemistry 24th ed. p. 478
MPL: 0.25

546. The chromosomes of mammalian cells are 20 times as large as those of Escherichia coli.
How can replication of mammalian chromosomes be carried out in just a few minutes?
A. The higher temperature of mammalian cells allows for an exponentially higher
replication rate.

Page 60 of 71
Page 61 of 71
Biochemistry

B. Hundreds of replication forks work simultaneously on each piece of chromosomal


DNA.
C. Many RNA polymerases carry out replication simultaneously on chromosomal DNA.
D. The presence of histones speeds up the rate of chromosomal DNA replication.
Ans. B
Ref: Harper’s Biochemistry 24th ed. p. 407
MPL: 0.75

547. Following ultraviolet damage to DNA in the skin, which of the following is MOST likely to
occur?
A. A specific nuclease detects damaged areas.
B. Purine dimmers are formed.
C. Both strands are cleaved.
D. Endonuclease removes the damaged strand.

Ref: Harper’s Biochemistry 24th ed. p. 814


MPL: 0.75

548. Which of the following statements about the genetic code is the MOST accurate?
A. Information is stored as sets of dinucleotide repeats called codons.
B. The code is degenerate.
C. There are 64 codons, all of which code for amino acids.
D. Information is stored as sets of trinucleotide repeats called codons.

Ref: Harper’s Biochemistry 24th ed. p. 433-434


MPL: 0.75

549. Which of the following is provided by the Northern Blot Analysis?


A. Detection of specific base pairs.
B. Detection of DNA molecules.
C. Detection of RNA molecules.
D. Detection of proteins.

Ref: Harper’s Biochemistry 24th ed. p. 472-473


MPL: 1.0

550. In contrast to DNA polymerase, RNA polymerase:


A. Fills in the gap between the Okazaki fragments.
B. Works only in a 5’ to 3’ direction.
C. Edits as it synthesizes.
D. Synthesizes RNA primer to initiate DNA synthesis.

Ref: Harper’s Biochemistry 24th ed. p. 418


MPL: 0.75

551. What is the function of a promoter site on DNA?


A. Transcribes repressor
B. Initiates transcription
C. Codes for RNA polymerase
D. Regulates termination

Ref: Harper’s Biochemistry 24th ed. p. 420


MPL: 1.0

552. The consensus sequence 5’-TATAAAA-3’ found ineukaryotic genes is almost similar to a
consensus sequence observed in prokaryotes. The consensus sequence is important as the:
A. Only site of binding of RNA polymerase II.
B. Promoter site for all RNA polymerases.
C. Termination site for RNA polymerase II.
D. First site of binding of a transcription factor for RNA polymerase II.
Ref: Harper’s Biochemistry 24th ed. pp. 421-422
MPL: 0.75
553. The so-called “caps” of RNA molecules:

Page 61 of 71
Page 62 of 71
Biochemistry

A. Allow tRNA to be processed.


B. Occur at the 3’-end of tRNA.
C. Are composed of poly A.
D. Are unique to eukaryotic mRNA.

Ref: Harper’s Biochemistry 24th ed. p. 428


MPL: 1.0

554. In bacterial RNA synthesis, what is the function of the “rho” factor?
A. It binds catabolite repressor to the promoter region.
B. It increases the rate of RNA synthesis.
C. It eliminates the binding of RNA polymerase to the promoter.
D. It participates in the proper termination of transcription.

Ref: Harper’s Biochemistry 24th ed. p. 419


MPL: 1.0

555. Which of the following statements correctly describes the nucleolus of a mammalian cell?
A. It differs from that found in bacterial cells in that histones are present.
B. It may contain hundreds of copies of genes for different types of ribosomal RNAs.
C. It synthesizes 5S rRNA.
D. It synthesizes all ribosomal RNA primary transcripts.

Ref; Harper’s Biochemistry 24th ed. p. 428


MPL: 0.75

556. Which of the following statements correctly describes the synthesis of mammalian mRNA?
A. Each mRNA often encodes several different proteins.
B. Several different genes may produce identical mRNA molecules.
C. Mammalian mRNA undergoes minimal modification during its maturation.
D. The RNA sequence transcribed from a gene is identical to the mRNA that exits from
nucleus to cytoplasm.

Ref: Harper’s Biochemistry 24th ed. p. 427


MPL: 0.75

557. Certain amino acids are not part of the primary structure of proteins but are modified after
translation. In scurvy, which amino acid that is normally part of collagen is not synthesized?
A. Hydroxytryptophan
B. Hydroxytyrosine
C. Hydroxyhistidine
D. Hydroxyproline

Ref: Harper’s Biochemistry 24th ed. pp. 668-669


MPL: 1,0

558. Immunoglobulin G (IgG) molecules can be characterized by which of the following


statements?
A. They are maintained at a constant level in the serum.
B. They contain nucleic acids.
C. They contain mostly carbohydrates.
D. They can be separated into subunits with a reducing agent and urea.

Ref: Harper’s Biochemistry 24th ed. p.716


MPL: 1.0

559. Which of the following proteolytic enzymes is activated by acid hydrolysis of the proenzyme
form?
A. Trypsin
B. Elastase
C. Pepsin
D. Carboxypeptidase

Ref: Harper’s Biochemistry 24th ed. p.636

Page 62 of 71
Page 63 of 71
Biochemistry

MPL: 1.0

560. Which of the following amino acids is ionizable in proteins?


A. Leucine
B. Histidine
C. Valine
D. Alanine

Ref: Harper’s Biochemistry 24th ed. p. 294


MPL: 0.75

561. The oxygen carrier of muscle is the globular protein myoglobin. Which of the following
amino acids is highly likely to be localized within the interior of the molecule?
A. Arginine
B. Aspartic acid
C. Glutamic acid
D. Valine

Ref: Harper’s Biochemistry 24th ed. p.53


MPL: 0.75

562. Which of the following statements concerning immunoglobulins is most accurate?


A. IgE is the principal antibody in the serum.
B. The heavy chains are similar in each class of immunoglobulin.
C. The constant regions of the heavy chains are the same in each class of
immunoglobulin.
D. IgE is the major immunoglobulin found in external secretions.

Ref: Harper’s Biochemistry 24th ed. p.718


MPL: 1.0

563. Under normal conditions in the blood, which of the following amino acid residues of
albumin is neutral?
A. Arginine
B. Aspartate
C. Glutamine
D. Histidine

Ref: Harper’s Biochemistry 24th ed. p. 294


MPL: 0.75

564. Which of the following carbohydrates would be most abundant in a diet of strict
vegetarians?
A. Amylose
B. Lactose
C. Cellulose
D. Glycogen

Ref: Harper’s Biochemistry 24th ed. p. 141


MPL: 1.0

565. The major metabolic product under normal circumstances by erythrocytes and by muscle
cells during intense exercise is recycled through the liver in the Cori cycle. The metabolite is:
A. Oxaloacetate
B. Glycerol
C. Alanine
D. Lactate

Ref: Harper’s Biochemistry 24th ed. p. 200-201


MPL: 0.75

Page 63 of 71
Page 64 of 71
Biochemistry

566. Following a fad diet of skimmed milk and yogurt, an adult female patient experiences
abdominal distention, nausea, cramping, and pain followed by watery diarrhea. This set of
symptoms is observed each time the said meal is consumed. A most likely diagnosis is:
A. Steatorrhea
B. Lactase deficiency
C. Sialidase deficiency
D. Lipoprotein deficiency

Ref: Harper’s Biochemistry 24th ed. p. 645


MPL: 0.1

567. Which of the following enzymes catalyzes the high-energy phosphorylation of substrates
during glycolysis?
A. Pyruvate kinase
B. Phosphoglycerate kinase
C. Triosephosphate isomerase
D. Glyceraldehyde-3-phosphate dehydrogenase

Ref: Harper’s Biochemistry 24th ed. p.176


MPL: 1.0

568. Which of the following enzymes is common to both glycolysis and gluconeogenesis?
A. Pyruvate carboxylase
B. Hexokinase
C. Phosphoglycerate kinase
D. Fructose 1,6-bisphosphatase

Ref: Harper’s Biochemistry 24th ed. p.194


MPL: 0.75

569. During the first week of a diet of 1500 calories per day, the oxidation of glucose via glycolysis
in the liver of a normal 59-kg woman is inhibited by the lowering of which of the following?
A. Citrate
B. ATP
C. Ketone bodies
D. Fructose-2,6-bisphosphatase

Ref: Harper’s Biochemistry 24th ed. p.176


MPL: 0.75

570. Familial fructokinase deficiency causes no symptoms because:


A. Hexokinase can phosphorylate fructose.
B. Most tissues utilize fructose.
C. Liver fructose-1-phosphate aldolase is still active.
D. Excess fructose spills into the bowel and is eliminated in the feces.

Ref: Harper’s Biochemistry 24th ed. p. 214


MPL: 0.75

571. Which of the following enzymes catalyzes phosphorylation with the use of inorganic
phosphate?
A. Hexokinase
B. Phosphofructokinase
C. Phosphoglycerate kinase
D. Glyceraldehyde-3-phosphate dehydrogenase

Ref: Harper’s Biochemistry 24th ed. p. 179


MPL: 0.75

572. After a well-rounded breakfast, which of the following would be expected to occur?
A. Increased activity of pyruvate carboxylase
B. Decreased activity of acetyl CoA carboxylase
C. Decreased rate of glycogenesis

Page 64 of 71
Page 65 of 71
Biochemistry

D. Decreased rate of protein synthesis

Ref. Harper’s Biochemistry 24th ed. p. 179


MPL: 0.75

573. Contraction of skeletal muscle is initiated by the binding of calcium to:


A. Tropomyosin
B. Troponin
C. Myosin
D. Actomyosin

Ref. Harper’s Biochemistry 24th ed. p. 687-688


MPL: 0.75

574. Which of the following statements correctly describes transport of oxygen by hemoglobin?
A. Oxygen binds to hemoglobin more avidly than does carbon monoxide.
B. The binding of oxygen to hemoglobin causes a valence change in the iron of the
heme moiety.
C. Each of the four heme moieties binds oxygen independently
D. The plot of percentage of oxygen bound versus oxygen pressure is sigmoid in
shape

Ref. Harper’s Biochemistry 24th ed. p. 436


MPL: 0.75

575. Which of the following mutations would produce a severe form of thalassemia?
A. Deletion of one α-globin locus
B. Deletion of one β-globin locus
C. Oxidation of heme groups to produce methemoglobin
D. Altered RNA processing at both

Ref. Harper’s Biochemistry 24th ed. p. 734


MPL 0.75

576. An increased affinity of hemoglobin for oxygen may result from which of the following?
A. Initial binding of oxygen to one of the four sites available in each
deoxyhemoglobin molecule
B. High pH
C. High carbon dioxide levels
D. High 2,3-bisphosphoglycerate levels within erthrocytes

Ref. Harper’s Biochemistry 24th ed. p. 436


MPL 0.75

577. The functions of many enzymes, membrane transporters, and other proteins can be quickly
activated or deactivated by phosphorylation of specific amino acid residues catalyzed by
enzymes called:
A. Cyclases
B. Kinases
C. Phosphatases
D. Proteases

Ref. Harper’s Biochemistry 24th ed. p. 64-65


MPL 0.75

578. The Vmax of the enzyme is the:


A. Reciprocal of the absolute value of the intercept of the curve with the x-axis
B. Reciprocal of the absolute value of the intercept of the curve with the y-axis
C. Absolute value of the intercept of the curve with the x-axis
D. Slope of the curve

Ref. Harper’s Biochemistry 24th ed. p. 75-90


MPL 0.75

Page 65 of 71
Page 66 of 71
Biochemistry

579. In the study of enzymes, the sigmoidal plot of substrate concentration versus reaction
velocity may indicate:
A. Michaelis-Menten Kinetics
B. Myoglobin binding oxygen
C. Cooperative binding
D. Competitive inhibition

Ref. Harper’s Biochemistry 24th ed. p. 75, 90


MPL 1.0

580. A non competitive inhibitor of an enzyme:


A. Increases Km with no or little change in Vmax
B. Decreases Km and decreases Vmax
C. Decreases Vmax
D. Increases Vmax

Ref. Harper’s Biochemistry 24th ed. p. 86-88


MPL 0.75

581. Which of the following statements correctly describes allosteric enzymes?


A. Effectors may enhance or inhibit substrate binding
B. They are not usually controlled by feedback inhibition
C. The regulatory site may be catalytic site
D. Michaelis-Menten kinetics describe their activity

Ref. Harper’s Biochemistry 24th ed. p. 104-105


MPL 0.75

582. Which of the following enzymes is regulated primarily through allosteric interaction?
A. Chymotrypsin
B. Pyruvate dehydrogenase
C. Glycogen phosphorylase
D. Aspartate transcarbamoylase

Ref. Harper’s Biochemistry 24th ed. p. 104-105


MPL 0.75

583. Which of the following statements about flavoproteins is TRUE?


A. They are not oxidized by coenzymeQ
B. They receive electrons from cytochrome P450 in liver mitochondria
C. They do not participate in oxidation of NADH dehydrogenises
D. They can be associated with sulfur and nonheme iron

Ref. Harper’s Biochemistry 24th ed. p. 117-118


MPL 0.75

584. Which of the following compounds is a member of the electron transport chain?
A. Octanoyl carnitine
B. Cytochrome c
C. NADH
D. Palmitoyl carnitine

Ref. Harper’s Biochemistry 24th ed. p. 128


MPL 0.75

585. All known effects of cyclic AMP in eukaryotic cells result from:
A. Activation of the catalytic unit of adenylate cyclase
B. Activation of synthetases
C. Activation of protein kinase
D. Stimulation of calcium release from the endoplasmic reticulum

Ref. Harper’s Biochemistry 24th ed. p. 165


MPL 0.75

Page 66 of 71
Page 67 of 71
Biochemistry

586. The connection between oxidative phosphorylation and electron transport is BEST described
by:
A. Existence of higher pH in the cisternae of the endoplasmic reticulum than in the
cytosol.
B. Synthesis of ATP as protons into the mitochondrial matrix along a proton gradient
that exist across the inner mitochondrial membrane
C. Dissociation of electron transport and oxidative phosphorylation
D. Absence of ATPase in the inner mitochondrial membrane

Ref. Harper’s Biochemistry 24th ed. p. 127


MPL 0.75

587. If all potential sources of ATP production are taken into account, the net number of ATP
molecules formed per molecule of glucose in aerobic glycolysis is:
A. 2
B. 6
C. 18
D. 36

Ref. Harper’s Biochemistry 24th ed. p. 176-184


MPL 0.75

588. Which of the following reactions generates ATP/


A. Glucose 6-phosphate to Fructose 6-phosphate
B. Glucose to Glucose 6-Phosphate
C. Fructose 6-phosphate to Fructose 1,6-bisphosphate
D. Phosphoenolpyruvate

Ref. Harper’s Biochemistry 24th ed. p. 176-184


MPL 0.75

589. Which of the following products of triacylglycerol breakdown and subsequent β-oxidation
may undergo gluconeogenesis?
A. Propionyl CoA
B. Acetyl CoA
C. All ketone bodies
D. Some amino acids

Ref. Harper’s Biochemistry 24th ed. p. 284


MPL 0.75

590. Inhibition of ATP synthesis during oxidative phosphorylation by oligomycin is thought to be


due to:
A. Blocking of the proton gradient between NADH-Q reductase & QH2
B. Blocking of the proton gradient between cytochrome C1 and cytochrome C
C. Dissociation of cytochrome c from mitochondrial membranes
D. Inhibition of mitochondrial ATPase

Ref. Harper’s Biochemistry 24th ed. p. 701


MPL 0.75

591. The reactions of the urea cycle occur:


A. In the cytosol
B. In the mitochondrial matrix
C. In the mitochondrial matrix & the cytosol
D. Only in lysosomes

Ref. Harper’s Biochemistry 24th ed. p. 301-380


MPL 0.75

592. The thyroid hormone thyroxine (T4) is derived from:


A. Threonine
B. Tyrosine

Page 67 of 71
Page 68 of 71
Biochemistry

C. Thiamine
D. Tryptophan

Ref. Harper’s Biochemistry 24th ed. p. 534-536


MPL 0.75

593. Which of the metabolites below is a precursor of tyrosine?


A. L-dihydroxyphenylalanine (DOPA)
B. Dopamine
C. Norepinephrine
D. Phenylalanine

Ref. Harper’s Biochemistry 24th ed. p. 296-297


MPL 0.75

594. Which clinical laboratory observation is suggestive of Hartnup’s disease?


A. High fecal levels of tryptophan and indole derivatives
B. Elevated plasma tyrosine and methionine levels
C. Elevation of glutamine in blood and urine
D. Extremely high levels of citrulline in urine

Ref. Harper’s Biochemistry 24th ed. p. 322-325


MPL 0.75

595. The important receive group of glutathione in its role as antioxidant is:
A. Serine
B. Sulfhydryl
C. Tyrosine
D. Acetyl CoA

Ref. Harper’s Biochemistry 24th ed. p. 753


MPL 0.75

596. A newborn develops jaundice that requires laboratory evaluation. Which of the following
porphyrin derivatives is conjugated, reacts directly, and is a major component of bile?
A. Stercobilin
B. Biliverdin
C. Bilirubin
D. Bilirubin diglucuronide

Ref. Harper’s Biochemistry 24th ed. p. 356


MPL 0.75

597. Which of the following poryphrins give stools their characteristic brown color?
A. Biliverdin
B. Urobilinogen
C. Heme
D. Stercobilin

Ref. Harper’s Biochemistry 24th ed. p. 354


MPL 0.75

598. Ceramide is a precursor to which of the following compounds?


A. Phosphatidyl serine
B. Sphingomyelin
C. Phosphatidyl glycerol
D. Phosphatidyl choline

Ref. Harper’s Biochemistry 24th ed. p. 250-252


MPL 0.75

599. Humans MOST easily tolerate a lack of which of the following nutrients?
A. Protein

Page 68 of 71
Page 69 of 71
Biochemistry

B. Iodine
C. Carbohydrate
D. Lipid

Ref. Harper’s Biochemistry 24th ed. p. 285-286


MPL 0.75

600. A deficiency in Vitamin B12 causes:


A. Cheilosis
B. Beriberi
C. Pernicious anemia
D. Scurvy

Ref. Harper’s Biochemistry 24th ed. p. 612-613


MPL 0.75

601. Which of the following vitaminswould MOST likely become deficient in a person who
develops a completely carnivorous lifestyle?
A. Thiamine
B. Niacin
C. Cobalamin
D. Vitamin C

Ref. Harper’s Biochemistry 24th ed. p. 612-613


MPL 0.75

602. Biotin is involved in which of the following types of reactions?


A. Hydroxlations
B. Carboxylations
C. Decarboxylations
D. Dehydrations

Ref. Harper’s Biochemistry 24th ed. p. 606-607


MPL 0.75
603. Which of the following conditions would primarily produce a functional deficiency of
Vitamin K?
A. Coumadin Therapy
B. Broad Spectrum antibiotic
C. Premature birth
D. Lack of red meat in the diet

Ref. Harper’s Biochemistry 24th ed. p. 727


MPL 0.75

604. A 3 month old boy presents with poor feeding & growth, hypotonia, lactic acidemia, and
mild acidosis. The pyruvate to lactate ratio is high, and there is decreased conversion of
pyruvate to Acetyl CoA. Which of the following may be considered for therapy?
A. Thiamine
B. FFA
C. Biotin
D. Vitamin C

Ref. Harper’s Biochemistry 24th ed. p. 183


MPL 0.25

605. A 2-year old child presents with cough and bronchitis, growth failure, & chronic diarrhea
with light-colored foul smelling stools. A deficiency of which vitamin should be considered?
A. Vit B6
B. Vit C
C. Vit A
D. Vit B1

Ref. Harper’s Biochemistry 24th ed. p. 614-617

Page 69 of 71
Page 70 of 71
Biochemistry

MPL 0.25

606. An infant presents with prominent forehead, bowing of the limbs, broad & tender wrists,
swelling at the costochondral junction of the ribs, and irritability. Which of the following
treatments is the most appropriate for this patient?
A. Removal of eggs from diet
B. Milk & Sunlight exposure
C. Diet of baby food containing liver & ground beef
D. Diet of baby food containing leafy vegetables

Ref. Harper’s Biochemistry 24th ed. p. 613


MPL 0.25

607. Allopurinol effectively treats gout but has NO effect on the severe neurological symptoms
of Lesch-Nyhan patients because it does not:
A. Decrease de Novo purine synthesis
B. Decrease de Novo Pyrimidine syntheis
C. Inhibit xanthine oxidase
D. Increase PRPP levels

Ref. Harper’s Biochemistry 24th ed. p. 378


MPL 0.75

608. Which of the following would rule out hyperuricemia in a ptient?


A. Lesch-Nyhan synthesis
B. Gout
C. Carbonage phosphate deficiency
D. Xanthine oxidase hyperactivity

Ref. Harper’s Biochemistry 24th ed. p. 381


MPL 0.75

609. Which of the following contributes nitrogen atom to both purine & pyrimidine rings?
A. Aspartate
B. Carbonage PO4
C. CO2
D. None of the above

Ref. Harper’s Biochemistry 24th edition P. 376-377


MPL 1.0

610. What is the end product of fatty acid synthase activity in human?
A. Palmitic acid
B. Palmetoleic acid
C. Linoleic
D. Arachidonic acid

Ref. Harper’s Biochemistry 24th ed. p. 216-223


MPL 0.75

611. For every 2 moles of free glycerol released by lipolysis of triglycerides in adipose tissue?
A. 1 mole of glucose can be synthesized via gluconeogenesis
B. 2 moles of triacrylglyceride is released
C. 2 moles of FFA is released
D. 3 moles of Acyl CoA is produced

Ref. Harper’s Biochemistry 24th ed. p. 267


MPL 0.75

612. A 45 year old man ha s a mild heart attack and is placed on diet and Mevastatin therapy.
Which of the following will result in this therapy?
A. Low blood glucose level

Page 70 of 71
Page 71 of 71
Biochemistry

B. Low blood LDLs


C. High blood cholesterol
D. Low oxidation of Fa

Ref. Harper’s Biochemistry 24th ed. p. 281


MPL 0.75

Page 71 of 71

Potrebbero piacerti anche